Download as docx, pdf, or txt
Download as docx, pdf, or txt
You are on page 1of 28

SET 2 such as formulating the care of plan, intervention and

evaluating the care during her shift. At the end of her shift,
PNLE I Nursing Practice the RN will pass this same task to the next RN in charge. This
Scope of this Nursing Test I is parallel to the NP1 NLE nursing care illustrates of what kind of method?
Coverage:
Foundation of Nursing primary nursing method
Nursing Research case method
Professional Adjustment team method
Leadership and Management functional method
7. A newly hired nurse on an adult medicine unit with 3
1. The registered nurse is planning to delegate tasks to months experience was asked to float to pediatrics. The
unlicensed assistive personnel (UAP). Which of the following nurse hesitates to perform pediatric skills and receive an
task could the registered nurse safely assigned to a UAP? interesting assignment that feels overwhelming. The nurse
should:
Monitor the I&O of a comatose toddler client with salicylate
poisoning resign on the spot from the nursing position and apply for a
Perform a complete bed bath on a 2-year-old with multiple position that does not require floating
injuries from a serious fall Inform the nursing supervisor and the charge nurse on the
Check the IV of a preschooler with Kawasaki disease pediatric floor about the nurse’s lack of skill and feelings of
Give an outmeal bath to an infant with eczema hesitations and request assistance
2. A nurse manager assigned a registered nurse from Ask several other nurses how they feel about pediatrics and
telemetry unit to the pediatrics unit. There were three find someone else who is willing to accept the assignment
patients assigned to the RN. Which of the following patients Refuse the assignment and leave the unit requesting a
should not be assigned to the floated nurse? vacation a day
8. An experienced nurse who voluntarily trained a less
A 9-year-old child diagnosed with rheumatic fever experienced nurse with the intention of enhancing the skills
A young infant after pyloromyotomy and knowledge and promoting professional advancement to
A 4-year-old with VSD following cardiac catheterization the nurse is called a:
A 5-month-old with Kawasaki disease
3. A nurse in charge in the pediatric unit is absent. The nurse mentor
manager decided to assign the nurse in the obstetrics unit to team leader
the pediatrics unit. Which of the following patients could the case manager
nurse manager safely assign to the float nurse? change agent
9. The pediatrics unit is understaffed and the nurse manager
A child who had multiple injuries from a serious vehicle informs the nurses in the obstetrics unit that she is going to
accident assign one nurse to float in the pediatric units. Which
A child diagnosed with Kawasaki disease and with cardiac statement by the designated float nurse may put her job at
complications risk?
A child who has had a nephrectomy for Wilm’s tumor
A child receiving an IV chelating therapy for lead poisoning “I do not get along with one of the nurses on the pediatrics
4. The registered nurse is planning to delegate task to a unit”
certified nursing assistant. Which of the following clients “I have a vacation day coming and would like to take that
should not be assigned to a CAN? now”
“I do not feel competent to go and work on that area”
A client diagnosed with diabetes and who has an infected toe “ I am afraid I will get the most serious clients in the unit”
A client who had a CVA in the past two months 10. The newly hired staff nurse has been working on a
A client with Chronic renal failure medical unit for 3 weeks. The nurse manager has posted the
A client with chronic venous insufficiency team leader assignments for the following week. The new
5. The nurse in the medication unit passes the medications staff knows that a major responsibility of the team leader is
for all the clients on the nursing unit. The head nurse is to:
making rounds with the physician and coordinates clients’
activities with other departments. The nurse assistant Provide care to the most acutely ill client on the team
changes the bed lines and answers call lights. A second nurse Know the condition and needs of all the patients on the team
is assigned for changing wound dressings; a licensed Document the assessments completed by the team members
practitioner nurse takes vital signs and bathes theclients. This Supervise direct care by nursing assistants
illustrates of what method of nursing care? 11. A 15-year-old girl just gave birth to a baby boy who needs
emergency surgery. The nurse prepared the consent form
Case management method and it should be signed by:
Primary nursing method
Team method The Physician
Functional method The Registered Nurse caring for the client
6. A registered nurse has been assigned to six clients on the The 15-year-old mother of the baby boy
12-hour shift. The RN is responsible for every aspect of care The mother of the girl
12. A nurse caring to a client with Alzheimer’s disease want any surgery or treatment because of religious beliefs
overheard a family member say to the client, “if you pee one about reincarnation. What is the role of the RN?
more time, I won’t give you any more food and drinks”. What
initial action is best for the nurse to take? call a family meeting
discuss the religious beliefs with the physician
Take no action because it is the family member saying that to encourage the client to have the surgery
the client inform the client of other options
Talk to the family member and explain that what she/he has 17. While in the hospital lobby, the RN overhears the three
said is not appropriate for the client staff discussing the health condition of her client. What
Give the family member the number for an Elder Abuse Hot would be the appropriate nursing action for the RN to take?
line
Document what the family member has said Tell them it is not appropriate to discuss the condition of the
13. Which is true about informed consent? client
Ignore them, because it is their right to discuss anything they
A nurse may accept responsibility signing a consent form if want to
the client is unable Join in the conversation, giving them supportive input about
Obtaining consent is not the responsibility of the physician the case of the client
A physician will not subject himself to liability if he withholds Report this incident to the nursing supervisor
any facts that are necessary to form the basis of an intelligent 18. A staff nurse has had a serious issue with her colleague. In
consent this situation, it is best to:
If the nurse witnesses a consent for surgery, the nurse is, in
effect, indicating that the signature is that of the purported Discuss this with the supervisor
person and that the person’s condition is as indicated at the Not discuss the issue with anyone. It will probably resolve
time of signing itself
14. A mother in labor told the nurse that she was expecting Try to discuss with the colleague about the issue and resolve
that her baby has no chance to survive and expects that the it when both are calmer
baby will be born dead. The mother accepts the fate of the Tell other members of the network what the team member
baby and informs the nurse that when the baby is born and did
requires resuscitation, the mother refuses any treatment to 19. The nurse is caring to a client who just gave birth to a
her baby and expresses hostility toward the nurse while the healthy baby boy. The nurse may not disclose confidential
pediatric team is taking care of the baby. The nurse is legally information when:
obligated to:
The nurse discusses the condition of the client in a clinical
Notify the pediatric team that the mother has refused conference with other nurses
resuscitation and any treatment for the baby and take the The client asks the nurse to discuss the her condition with the
baby to the mother family
Get a court order making the baby a ward of the court The father of a woman who just delivered a baby is on the
Record the statement of the mother, notify the pediatric phone to find out the sex of the baby
team, and observe carefully for signs of impaired bonding A researcher from an institutionally approved research study
and neglect as a reasonable suspicion of child abuse reviews the medical record of a patient
Do nothing except record the mother’s statement in the 20. A 17-year-old married client is scheduled for surgery. The
medical record nurse taking care of the client realizes that consent has not
15. The hospitalized client with a chronic cough is scheduled been signed after preoperative medications were given.
for bronchoscopy. The nurse is tasks to bring the informed What should the nurse do?
consent document into the client’s room for a signature. The
client asks the nurse for details of the procedure and Call the surgeon
demands an explanation why the process of informed Ask the spouse to sign the consent
consent is necessary. The nurse responds that informed Obtain a consent from the client as soon as possible
consent means: Get a verbal consent from the parents of the client
21. A 12-year-old client is admitted to the hospital. The
The patient releases the physician from all responsibility for physician ordered Dilantin to the client. In administering IV
the procedure. phenytoin (Dilantin) to a child, the nurse would be most
The immediate family may make decision against the correct in mixing it with:
patient’s will.
The physician must give the client or surrogates enough Normal Saline
information to make health care judgments consistent with Heparinized normal saline
their values and goals. 5% dextrose in water
The patient agrees to a procedure ordered by the physician Lactated Ringer’s solution
even if the client does not understand what the outcome will 22. The nurse is caring to a client who is hypotensive.
be. Following a large hematemesis, how should the nurse
16. A hospitalized client with severe necrotizing ulcer of the position the client?
lower leg is schedule for an amputation. The client tells the
nurse that he will not sign the consent form and he does not
Feet and legs elevated 20 degrees, trunk horizontal, head on Facial edema with ecchymosis and handprint mark: crackles
small pillow and wheezes
Low Fowler’s with knees gatched at 30 degrees Facial edema, with red marks; crackles in the lung
Supine with the head turned to the left Facial edema with ecchymosis that looks like a handprint
Bed sloped at a 45 degree angle with the head lowest and the Red bruise mark and ecchymosis on face
legs highest 30. On the evening shift, the triage nurse evaluates several
23. The client is brought to the emergency department after clients who were brought to the emergency department.
a serious accident. What would be the initial nursing action of Which in the following clients should receive highest priority?
the nurse to the client?
an elderly woman complaining of a loss of appetite and
assess the level of consciousness and circulation fatigue for the past week
check respirations, circulation, neurological response A football player limping and complaining of pain and
align the spine, check pupils, check for hemorrhage swelling in the right ankle
check respiration, stabilize spine, check circulation A 50-year-old man, diaphoretic and complaining of severe
24. A nurse is assigned to care to a client with Parkinson’s chest pain radiating to his jaw
disease. What interventions are important if the nurse wants A mother with a 5-year-old boy who says her son has been
to improve nutrition and promote effective swallowing of the complaining of nausea and vomited once since noon
client? 31. A 80-year-old female client is brought to the emergency
department by her caregiver, on the nurse’s assessment; the
Eat solid food following are the manifestations of the client: anorexia,
Give liquids with meals cachexia and multiple bruises. What would be the best
Feed the client nursing intervention?
Sit in an upright position to eat
25. During tracheal suctioning, the nurse should implement check the laboratory data for serum albumin, hematocrit,
safety measures. Which of the following should the nurse and hemoglobin
implements? talk to the client about the caregiver and support system
complete a police report on elder abuse
limit suction pressure to 150-180 mmHg complete a gastrointestinal and neurological assessment
suction for 15-20 seconds 32. The night shift nurse is making rounds. When the nurse
wear eye goggles enters a client’s room, the client is on the floor next to the
remove the inner cannula bed. What would be the initial action of the nurse?
26. The nurse is conducting a discharge instructions to a
client diagnosed with diabetes. What sign of hypoglycemia chart that the patient fell
should be taught to a client? call the physician
chart that the client was found on the floor next to the bed
warm, flushed skin fill out an incident report
hunger and thirst 33. The nurse on the night shift is about to administer
increase urinary output medication to a preschooler client and notes that the child
palpitation and weakness has no ID bracelet. The best way for the nurse to identify the
27. A client admitted to the hospital and diagnosed with client is to ask:
Addison’s disease. What would be the appropriate nursing
action to the client? The adult visiting, “The child’s name is
____________________?”
administering insulin-replacement therapy The child, “Is your name____________?”
providing a low-sodium diet Another staff nurse to identify this child
restricting fluids to 1500 ml/day The other children in the room what the child’s name is
reducing physical and emotional stress 34. The nurse caring to a client has completed the
28. The nurse is to perform tracheal suctioning. During assessment. Which of the following will be considered to be
tracheal suctioning, which nursing action is essential to the most accurate charting of a lump felt in the right breast?
prevent hypoxemia?
“abnormally felt area in the right breast, drainage noted”
aucultating the lungs to determine the baseline data to “hard nodular mass in right breast nipple”
assess the effectiveness of suctioning “firm mass at five ‘ clock, outer quadrant, 1cm from right
removing oral and nasal secretions nipple’
encouraging the patient to deep breathe and cough to “mass in the right breast 4cmx1cm
facilitate removal of upper-airway secretions 35. The physician instructed the nurse that intravenous
administering 100% oxygen to reduce the effects of airway pyelogram will be done to the client. The client asks the
obstruction during suctioning. nurse what is the purpose of the procedure. The appropriate
29. An infant is admitted and diagnosed with pneumonia and nursing response is to:
suspicious-looking red marks on the swollen face resembling
a handprint. The nurse does further assessment to the client. outline the kidney vasculature
How would the nurse document the finding? determine the size, shape, and placement of the kidneys
test renal tubular function and the patency of the urinary 42. After a birth, the physician cut the cord of the baby, and
tract before the baby is given to the mother, what would be the
measure renal blood flow initial nursing action of the nurse?
36. A client visits the clinic for screening of scoliosis. The
nurse should ask the client to: examine the infant for any observable abnormalities
confirm identification of the infant and apply bracelet to
bend all the way over and touch the toes mother and infant
stand up as straight and tall as possible instill prophylactic medication in the infant’s eyes
bend over at a 90-degree angle from the waist wrap the infant in a prewarmed blanket and cover the head
bend over at a 45-degree angle from the waist 43. A 2-year-old client is admitted to the hospital with severe
37. A client with tuberculosis is admitted in the hospital for 2 eczema lesions on the scalp, face, neck and arms. The client is
weeks. When a client’s family members come to visit, they scratching the affected areas. What would be the best
would be adhering to respiratory isolation precautions when nursing intervention to prevent the client from scratching the
they: affected areas?

wash their hands when leaving elbow restraints to the arms


put on gowns, gloves and masks Mittens to the hands
avoid contact with the client’s roommate Clove-hitch restraints to the hands
keep the client’s room door open A posey jacket to the torso
38. An infant is brought to the emergency department and 44. The parents of the hospitalized client ask the nurse how
diagnosed with pyloric stenosis. The parents of the client ask their baby might have gotten pyloric stenosis. The
the nurse, “Why does my baby continue to vomit?” Which of appropriate nursing response would be:
the following would be the best nursing response of the
nurse? There is no way to determine this preoperatively
Their baby was born with this condition
“Your baby eats too rapidly and overfills the stomach, which Their baby developed this condition during the first few
causes vomiting weeks of life
“Your baby can’t empty the formula that is in the stomach Their baby acquired it due to a formula allergy
into the bowel” 45. A male client comes to the clinic for check-up. In doing a
“The vomiting is due to the nausea that accompanies pyloric physical assessment, the nurse should report to the physician
stenosis” the most common symptom of gonorrhea, which is:
“Your baby needs to be burped more thoroughly after
feeding” pruritus
39. A 70-year-old client with suspected tuberculosis is pus in the urine
brought to the geriatric care facilities. An intradermal WBC in the urine
tuberculosis test is schedule to be done. The client asks the Dysuria
nurse what is the purpose of the test. Which of the following 46. Which of the following would be the most important goal
would be the best rationale for this? in the nursing care of an infant client with eczema?

reactivation of an old tuberculosis infection preventing infection


increased incidence of new cases of tuberculosis in persons maintaining the comfort level
over 65 years old providing for adequate nutrition
greater exposure to diverse health care workers decreasing the itching
respiratory problems are characteristic in this population 47. The nurse is making a discharge instruction to a client
40. The nurse is making a health teaching to the parents of receiving chemotherapy. The client is at risk for bone marrow
the client. In teaching parents how to measure the area of depression. The nurse gives instructions to the client about
induration in response to a PPD test, the nurse would be how to prevent infection at home. Which of the following
most accurate in advising the parents to measure: health teaching would be included?

both the areas that look red and feel raised “Get a weekly WBC count”
The entire area that feels itchy to the child “Do not share a bathroom with children or pregnant woman”
Only the area that looks reddened “Avoid contact with others while receiving chemotherapy”
Only the area that feels raised “Do frequent hand washing and maintain good hygiene”
41. A community health nurse is schedule to do home visit. 48. The nurse is assigned to care the client with infectious
She visits to an elderly person living alone. Which of the disease. The best antimicrobial agent for the nurse to use in
following observation would be a concern? handwashing is:

Picture windows Isopropyl alcohol


Unwashed dishes in the sink Hexachlorophene (Phisohex)
Clear and shiny floors Soap and water
Brightly lit rooms Chlorhexidine gluconate (CHG) (Hibiclens)
49. The mother of the client tells the nurse, “ I’m not going to 14. C. Although the statements by the mother may not
have my baby get any immunization”. What would be the create a suspicion of neglect, when they are coupled
best nursing response to the mother? with observations about impaired bonding and
maternal attachment, they may impose the obligation
“You and I need to review your rationale for this decision” to report child neglect. The nurse is further obligated
“Your baby will not be able to attend day care without to notify caregivers of refusal to consent to treatment
immunizations” 15. C. It best explains what informed consent is and
“Your decision can be viewed as a form of child abuse and provides for legal rights of the patient
neglect” 16. B. The physician may not be aware of the role that
“You are needlessly placing other people at risk for religious beliefs play in making a decision about
communicable diseases” surgery.
50. The nurse is teaching the client about breast self- 17. A. The behavior should be stopped. The first step is to
examination. Which observation should the client be taught remind the staff that confidentiality may be violated
to recognize when doing the examination for detection of 18. C. Waiting for emotions to dissipate and sitting down
breast cancer? with the colleague is the first rule of conflict
resolution.
tender, movable lump 19. C. The nurse has no idea who the person is on the
pain on breast self-examination phone and therefore may not share the information
round, well-defined lump even if the patient gives permission
dimpling of the breast tissue 20. A. The priority is to let the surgeon know, who in turn
may ask the husband to sign the consent.
Answers and Rationales 21. A. Phenytoin (Dilantin) can cause venous irritation due
1. D. Bathing an infant with eczema can be safely to its alkalinity, therefore it should be mixed with
delegated to an aide; this task is basic and can normal saline.
competently performed by an aid. 22. A. This position increases venous return, improves
2. B. The RN floated from the telemetry unit would be cardiac volume, and promotes adequate ventilation
least prepared to care for a young infant who has just and cerebral perfusion
had GI surgery and requires a specific feeding regimen. 23. D. Checking the airway would be a priority, and a neck
3. C. RN floated from the obstetrics unit should be able to injury should be suspected
care for a client with major abdominal surgery, 24. D. Client with Parkinson’s disease are at a high risk for
because this nurse has experienced caring for clients aspiration and undernutrition. Sitting upright
with cesarean births. promotes more effective swallowing.
4. A. The patient is experiencing a potentially serious 25. C. It is important to protect the RN’s eyes from the
complication related to diabetes and needs ongoing possible contamination of coughed-up secretions
assessment by an RN 26. D. There has been too little food or too much insulin.
5. D. It describes functional nursing. Staff is assigned to Glucose levels can be markedly decreased (less than 50
specific task rather than specific clients. mg/dl). Severe hypoglycemia may be fatal if not
6. B. Case management. The nurse assumes total detected
responsibility for meeting the needs of the client 27. D. Because the client’s ability to react to stress is
during her entire duty. decreased, maintaining a quiet environment becomes
7. B. The nurse is ethically obligated to inform the person a nursing priority. Dehydration is a common problem
responsible for the assignment and the person in Addison’s disease, so close observation of the
responsible for the unit about the nurse’s skill level. client’s hydration level is crucial.
The nurse therefore avoids a situation of 28. D. Presuctioning and postsuctioning ventilation with
abandoningclients and exposing them to greater risks 100% oxygen is important in reducing hypoxemia
8. A. This describes a mentor which occurs when the flow of gases in the airway is
9. B. This action demonstrates a lack of responsibility and obstructed by the suctioning catheter.
the nurse should attempt negotiation with the nurse 29. B. This is an example of objective data of both
manager. pulmonary status and direct observation on the skin by
10. B. The team leader is responsible for the overall the nurse.
management of all clients and staff on the team, and 30. C. These are likely signs of an acute myocardial
this information is essential in order to accomplish this infarction (MI). An acute MI is a cardiovascular
11. C. Even though the mother is a minor, she is legally emergency requiring immediate attention. Acute MI is
able to sign consent for her own child. potentially fatal if not treated immediately.
12. B. This response is the most direct and immediate. 31. D. Assessment and more data collection are needed.
This is a case of potential need for advocacy and The client may have gastrointestinal or neurological
patient’s rights. problems that account for the symptoms. The anorexia
13. D. The nurse who witness a consent for treatment or could result from medications, poor dentition, or
surgery is witnessing only that the client signed the indigestion, and the bruises may be attributed to
form and that the client’s condition is as indicated at ataxia, frequent falls, vertigo or medication.
the time of signing. The nurse is not witnessing that 32. B. This is closest to suggesting action-assessment,
the client is “informed”. rather than paperwork- and is therefore the best of the
four.
33. C. The only acceptable way to identify a preschooler misconceptions regarding immunizations that may
client is to have a parent or another staff member exist.
identify the client. 50. D. The tumor infiltrates nearby tissue, it can cause
34. C. It describes the mass in the greatest detail. retraction of the overlying skin and create a dimpling
35. C. Intravenous pyelogram tests both the function and appearance.
patency of the kidneys. After the intravenous injection
of a radiopaque contrast medium, the size, location, PNLE II Nursing Practice
and patency of the kidneys can be observed by The scope of this Nursing Test II is parallel to the NP2 NLE
roentgenogram, as well as the patency of the urethra Coverage:
and bladder as the kidneys function to excrete the Maternal and Child Health
contrast medium. Community Health Nursing
36. C. This is the recommended position for screening for Communicable Diseases
scoliosis. It allows the nurse to inspect the alignment Integrated Management of Childhood Illness
of the spine, as well as to compare both shoulders and
both hips. 1. The student nurse is assigned to take the vital signs of the
37. A. Handwashing is the best method for reducing cross- clients in the pediatric ward. The student nurse reports to the
contamination. Gowns and gloves are not always staff nurse that the parent of a toddler who is 2 days
required when entering a client’s room. postoperative after a cleft palate repair has given the toddler
38. B. Pyloric stenosis is an anomaly of the upper a pacifier. What would be the best immediate action of the
gastrointestinal tract. The condition involves a nurse?
thickening, or hypertrophy, of the pyloric sphincter
located at the distal end of the stomach. This causes a Notify the pediatrician of this finding
mechanical intestinal obstruction, which leads to Reassure the student that this is an acceptable action on the
vomiting after feeding the infant. The vomiting parent’s part
associated with pyloric stenosis is described as being Discuss this action with the parents
projectile in nature. This is due to the increasing Ask the student nurse to remove the pacifier from the
amounts of formula the infant begins to consume toddler’s mouth
coupled with the increasing thickening of the pyloric 2. The nurse is providing a health teaching to the mother of
sphincter. an 8-year-old child with cystic fibrosis. Which of the following
39. B. Increased incidence of TB has been seen in the statement if made by the mother would indicate to the nurse
general population with a high incidence reported in the need for further teaching about the medication regimen
hospitalized elderly clients. Immunosuppression and of the child?
lack of classic manifestations because of the aging
process are just two of the contributing factors of “My child might need an extra capsule if the meal is high in
tuberculosis in the elderly. fat”
40. D. Parents should be taught to feel the area that is “I’ll give the enzyme capsule before every snack”
raised and measure only that. “I’ll give the enzyme capsule before every meal”
41. C. It is a safety hazard to have shiny floors because “My child hates to take pills, so I’ll mix the capsule into a cup
they can cause falls. of hot chocolate
42. D. The first priority, beside maintaining a newborn’s 3. The mother brought her child to the clinic for follow-up
patent airway, is body temperature. check up. The mother tells the nurse that 14 days after
43. B. The purpose of restraints for this child is to keep starting an oral iron supplement, her child’s stools are black.
the child from scratching the affected areas. Mittens Which of the following is the best nursing response to the
restraint would prevent scratching, while allowing the mother?
most movement permissible.
44. C. Pyloric stenosis is not a congenital anatomical “I will notify the physician, who will probably decrease the
defect, but the precise etiology is unknown. It dosage slightly”
develops during the first few weeks of life. “This is a normal side effect and means the medication is
45. B. Pus is usually the first symptom, because the working”
bacteria reproduce in the bladder. “You sound quite concerned. Would you like to talk about
46. A. Preventing infection in the infant with eczema is the this further?”
nurse’s most important goal. The infant with eczema is “I will need a specimen to check the stool for possible
at high risk for infection due to numerous breaks in the bleeding”
skin’s integrity. Intact skin is always the infant’s first 4. An 8-year-old boy with asthma is brought to the clinic for
line of defense against infection. check up. The mother asks the nurse if the treatment given to
47. D. Frequent hand washing and good hygiene are the her son is effective. What would be the appropriate response
best means of preventing infection. of the nurse?
48. D. CHG is a highly effective antimicrobial ingredient,
especially when it is used consistently over time. I will review first the child’s height on a growth chart to know
49. A. The mother may have many reasons for such a if the treatment is working
decision. It is the nurse’s responsibility to review this I will review first the child’s weight on a growth chart to know
decision with the mother and clarify any if the treatment is working
I will review first the number of prescriptions refills the child 11. The nurse is caring to a 24-month-old child diagnosed
has required over the last 6 months to give you an accurate with congenital heart defect. The physician prescribed
answer digoxin (Lanoxin) to the client. Before the administration of
I will review first the number of times the child has seen the the drug, the nurse checks the apical pulse rate to be 110
pediatrician during the last 6 months to give you an accurate beats per minute and regular. What would be the next
answer nursing action?
5. The nurse is caring to a child client who is receiving
tetracycline. The nurse is aware that in taking this Check the other vital signs and level of consciousness
medication, it is very important to: Withhold the digoxin and notify the physician
Give the digoxin as prescribed
Administer the drug between meals Check the apical and radial simultaneously, and if they are
Monitor the child’s hearing the same, give the digoxin.
Give the drug through a straw 12. An 8-year-old client with cystic fibrosis is admitted to the
Keep the child out of the sunlight hospital and will undergo a chest physiotherapy treatment.
6. A 14 day-old infant with a cyanotic heart defects and mild The therapy should be properly coordinated by the nurse
congestive heart failure is brought to the emergency with the respiratory therapy department so that treatments
department. During assessment, the nurse checks the apical occur during:
pulse rate of the infant. The apical pulse rate is 130 beats per
minute. Which of the following is the appropriate nursing After meals
action? Between meals
After medication
Retake the apical pulse in 15 minutes Around the child’s play schedule
Retake the apical pulse in 30 minutes 13. The nurse is providing health teaching about the
Notify the pediatrician immediately breastfeeding and family planning to the client who gave
Administer the medication as scheduled birth to a healthy baby girl. Which of the following statement
7. The physician prescribed gentamicin (Garamycin) to a child would alert the nurse that the client needs further teaching?
who is also receiving chemotherapy. Before administering
the drug, the nurse should check the results of the child’s: “I understand that the hormones for breastfeeding may
affect when my periods come”
CBC and platelet count “Breastfeeding causes my womb to tighten and bleed less
Auditory tests after birth”
Renal Function tests “I may not have periods while I am breastfeeding, so I don’t
Abdominal and chest x-rays need family planning”
8. Which of the following is the suited size of the needle “I can get pregnant as early as one month after my baby was
would the nurse select to administer the IM injection to a born”
preschool child? 14. A toddler is brought to the hospital because of severe
diarrhea and vomiting. The nurse assigned to the client
18 G, 1-1/2 inch enters the client’s room and finds out that the client is using
25 G, 5/8 inch a soiled blanket brought in from home. The nurse attempts to
21 G, 1 inch remove the blanket and replace it with a new and clean
18 G, 1inch blanket. The toddler refuses to give the soiled blanket. The
9. A 9-year-old boy is admitted to the hospital. The boy is nurse realizes that the best explanation for the toddler’s
being treated with salicylates for the migratory polyarthritis behavior is:
accompanying the diagnosis of rheumatic fever. Which of the
following activities performed by the child would give a best The toddler did not bond well with the maternal figure
sign that the medication is effective? The blanket is an important transitional object
The toddler is anxious about the hospital experience
Listening to story of his mother The toddler is resistive to nursing interventions
Listening to the music in the radio 15. The nurse has knowledge about the developmental task
Playing mini piano of the child. In caring a 3-year-old-client, the nurse knows
Watching movie in the dvd mini player that the suited developmental task of this child is to:
10. The physician decided to schedule the 4-year-old client
for repair of left undescended testicle. The Injection of a Learn to play with other children
hormone, HCG finds it less successful for treatment. To Able to trust others
administer a pentobarbital sodium (Nembutal) suppository Express all needs through speaking
preoperatively to this client, in which position should the Explore and manipulate the environment
nurse place him? 16. A mother who gave birth to her second daughter is so
concerned about her 2-year old daughter. She tells the nurse,
Supine with foot of bed elevated “I am afraid that my 2-year-old daughter may not accept her
Prone with legs abducted newly born sister”. It is appropriate to the nurse to response
Sitting with foot of bed elevated that:
Side-lying with upper leg flexed
The older daughter be given more responsibility and assure Asks many questions regarding the condition and the
her “that she is a big girl now, and doesn’t need Mommy as procedure
much” Worry over the impending surgery
The older daughter not have interaction with the baby at the Be uninterested in the upcoming surgery
hospital, because she may harm her new sibling 23. The nine-year-old client is admitted in the hospital for
The older daughter stay with her grandmother for a few days almost 1 week and is on bed rest. The child complains of
until the parents and new baby are settled at home being bored and it seems tiresome to stay on bed and doing
The mother spend time alone with her older daughter when nothing. What activity selected by the nurse would the child
the baby is sleeping most likely find stimulating?
17. A 2-year-old client with cystic fibrosis is confined to bed
and is not allowed to go to the playroom. Which of the Watching a video
following is an appropriate toy would the nurse select for the Putting together a puzzle
child: Assembling handouts with the nurse for an upcoming staff
development meeting
Puzzle Listening to a compact disc
Musical automobile 24. The parent of a 16-year-old boy tells the nurse that his
Arranging stickers in the album son is driving a motorbike very fast and with one hand. “It is
Pounding board and hammer making me crazy!” What would be the best explanation of
18. Which of the following clients is at high risk for the nurse to the behavior of the boy?
developmental problem?
The adolescent might have an unconscious death wish
A toddler with acute Glomerulonephritis on antihypertensive The adolescent feels indestructible
and antibiotics The adolescent lacks life experience to realize how dangerous
A 5-year-old with asthma on cromolyn sodium the behavior is
A preschooler with tonsillitis The adolescent has found a way to act out hostility toward
A 2 1/2 –year old boy with cystic fibrosis the parent
19. Which of the following would be the best divesionary 25. An 8-month-old infant is admitted to the hospital due to
activity for the nurse to select for a 2 weeks hospitalized 3- diarrhea. The nurse caring for the client tells the mother to
year-old girl? stay beside the infant while making assessment. Which of the
following developmental milestones the infant has reached?
Crayons and coloring books
doll Has a three-word vocabulary
xylophone toy Interacts with other infants
puzzles Stands alone
20. A nurse is providing safety instructions to the parents of Recognizes but is fearful of strangers
the 11-month-old child. Which of the following will the nurse 26. The community nurse is conducting a health teaching in
includes in the instructions? the group of married women. When teaching a woman about
fertility awareness, the nurse should emphasize that the
Plugging all electrical outlets in the house basal body temperature:
Installing a gate at the top and bottom of any stairs in the
home Should be recorded each morning before any activity
Purchasing an infant car seat as soon as possible Is the average temperature taken each morning
Begin to teach the child not to place small objects in the Can be done with a mercury thermometer but not a digital
mouth one
21. An 8-year-old girl is in second grade and the parents Has a lower degree of accuracy in predicting ovulation than
decided to enroll her to a new school. While the child is the cervical mucus test
focusing on adjusting to new environment and peers, her 27. The community nurse is providing an instruction to the
grades suffer. The child’s father severely punishes the child clients in the health center about the use of diaphragm for
and forces her daughter to study after school. The father family planning. To evaluate the understanding of the
does not allow also her daughter to play with other children. woman, the nurse asks her to demonstrate the use of the
These data indicate to the nurse that this child is deprived of diaphragm. Which of following statement indicates a need
forming which normal phase of development? for further health teaching?

Heterosexual relationships “I should check the diaphragm carefully for holes every time I
A love relationship with the father use it.”
A dependency relationship with the father “The diaphragm must be left in place for at least 6 hours after
Close relationship with peers intercourse.”
22. A 5-year-old boy client is scheduled for hernia surgery. “I really need to use the diaphragm and jelly most during the
The nurse is preparing to do preoperative teaching with the middle of my menstrual cycle
child. The nurse should knows that the 5-year-old would: “I may need a different size diaphragm if I gain or lose more
than 20 pounds”
Expect a simple yet logical explanation regarding the surgery 28. The client visits the clinic for prenatal check-up. While
waiting for the physician, the nurse decided to conduct
health teaching to the client. The nurse informed the client statements is the appropriate nursing response to the
that primigravida mother should go to the hospital when mother?
which patter is evident?
“I will examine the child for symptoms of hepatitis B”
Contractions are 2-3 minutes apart, lasting 90 seconds, and “Your child will start the series again”
membranes have ruptured “Your child will get the next dose as soon as possible”
Contractions are 5-10 minutes apart, lasting 30 seconds, and “Your child will have a hepatitis titer done to determine if
are felt as strong menstrual cramps immunization has taken place.”
Contractions are 3-5 minutes apart, accompanied by rectal 34. The community health nurse implemented a new
pressure and bloody show program about effective breast cancer screening technique
Contractions are 5 minutes apart, lasting 60 seconds, and for the female personnel of the health department of
increasing in intensity Valenzuela. Which of the following technique should the
29. A nurse is planning a home visit program to a new mother nurse consider to be of the lowest priority?
who is 2 weeks postpartum and breastfeeding, the nurse
includes in her health teaching about the resumption of Yearly breast exam by a trained professional
fertility, contraception and sexual activity. Which of the Detailed health history to identify women at risk
following statement indicates that the mother has Screening mammogram every year for women over age 50
understood the teaching? Screening mammogram every 1-2 years for women over age
of 40.
“Because breastfeeding speeds the healing process after 35. Which of the following technique is considered an aseptic
birth, I can have sex right away and not worry about practice during the home visit of the community health
infection” nurse?
“Because I am breastfeeding and my hormones are
decreased, I may need to use a vaginal lubricant when I have Wrapping used dressing in a plastic bag before placing them
sex” in the nursing bag
“After birth, you have to have a period before you can get Washing hands before removing equipment from the nursing
pregnant again’ bag
“Breastfeeding protects me from pregnancy because it keeps Using the client’s soap and cloth towel for hand washing
my hormones down, so I don’t need any contraception until I Placing the contaminated needles and syringes in a labeled
stop breastfeeding” container inside the nursing bag
30. A community nurse enters the home of the client for 36. The nurse is planning to conduct a home visit in a small
follow-up visit. Which of the following is the most community. Which of the following is the most important
appropriate area to place the nursing bag of the nurse when factor when planning the best time for a home care visit?
conducting a home visit?
Purpose of the home visit
cushioned footstool Preference of the patient’s family
bedside wood table Location of the patient’s home
kitchen countertop Length of time of the visit will take
living room sofa 37. The nurse assigned in the health center is counseling a
31. The nurse in the health center is making an assessment to 30-year-old client requesting oral contraceptives. The client
the infant client. The nurse notes some rashes and small tells the nurse that she has an active yeast infection that has
fluid-filled bumps in the skin. The nurse suspects that the recurred several times in the past year. Which statement by
infant has eczema. Which of the following is the most the nurse is inaccurate concerning health promotion actions
important nursing goal: to prevent recurring yeast infection?

Preventing infection “During treatment for yeast, avoid vaginal intercourse for
Providing for adequate nutrition one week”
Decreasing the itching “Wear loose-fitting cotton underwear”
Maintaining the comfort level “Avoid eating large amounts of sugar or sugar-bingeing”
32. The nurse in the health center is providing immunization “Douche once a day with a mild vinegar and water solution”
to the children. The nurse is carefully assessing the condition 38. During immunization week in the health center, the
of the children before giving the vaccines. Which of the parent of a 6-month-old infant asks the health nurse, “Why is
following would the nurse note to withhold the infant’s our baby going to receive so many immunizations over a long
scheduled immunizations? time period?” The best nursing response would be:

a dry cough “The number of immunizations your baby will receive shows
a skin rash how many pediatric communicable and infectious diseases
a low-grade fever can now be prevented.”
a runny nose “You need to ask the physician”
33. A mother brought her child in the health center for “The number of immunizations your baby will receive is
hepatitis B vaccination in a series. The mother informs the determined by your baby’s health history and age”
nurse that the child missed an appointment last month to “It is easier on your baby to receive several immunizations
have the third hepatitis B vaccination. Which of the following rather than one at a time”
39. The community health nurse is conducting a health 44. The department of health is promoting the breastfeeding
teaching about nutrition to a group of pregnant women who program to all newly mothers. The nurse is formulating a plan
are anemic and are lactose intolerant. Which of the following of care to a woman who gave birth to a baby girl. The nursing
foods should the nurse especially encourage during the third care plan for a breast-feeding mother takes into account that
trimester? breast-feeding is contraindicated when the woman:

Cheese, yogurt, and fish for protein and calcium needs plus Is pregnant
prenatal vitamins and iron supplements Has genital herpes infection
Prenatal iron and calcium supplements plus a regular adult Develops mastitis
diet Has inverted nipples
Red beans, green leafy vegetables, and fish for iron and 45. The City health department conducted a medical mission
calcium needs plus prenatal vitamins and iron supplements in Barangay Marulas. Majority of the children in the Barangay
Red meat, milk and eggs for iron and calcium needs plus Marulas were diagnosed with pinworms. The community
prenatal vitamins and iron supplements health nurse should anticipate that the children’s chief
40. A woman with active tuberculosis (TB) and has visited the complaint would be:
health center for regular therapy for five months wants to
become pregnant. The nurse knows that further information Lack of appetite
is necessary when the woman states: Severe itching of the scalp
Perianal itching
“Spontaneous abortion may occur in one out of five women Severe abdominal pain
who are infected” 46. The mother brought her daughter to the health center.
“Pulmonary TB may jeopardize my pregnancy” The child has head lice. The nurse anticipates that the nursing
“I know that I may not be able to have close contact with my diagnosis most closely correlated with this is:
baby until contagious is no longer a problem
“I can get pregnant after I have been free of TB for 6 months” Fluid volume deficit related to vomiting
41. The Department of Health is alarmed that almost 33 Altered body image related to alopecia
million people suffer from food poisoning every year. Altered comfort related to itching
Salmonella enteritis is responsible for almost 4 million cases Diversional activity deficit related to hospitalization
of food poisoning. One of the major goals is to promote 47. The mother brings a child to the health care clinic
proper food preparation. The community health nurse is because of severe headache and vomiting. During the
tasks to conduct health teaching about the prevention of assessment of the health care nurse, the temperature of the
food poisoning to a group of mother everyday. The nurse can child is 40 degree Celsius, and the nurse notes the presence
help identify signs and symptoms of specific organisms to of nuchal rigidity. The nurse is suspecting that the child might
help patients get appropriate treatment. Typical symptoms of be suffering from bacterial meningitis. The nurse continues to
salmonella include: assess the child for the presence of Kernig’s sign. Which
finding would indicate the presence of this sign?
Nausea, vomiting and paralysis
Bloody diarrhea Flexion of the hips when the neck is flexed from a lying
Diarrhea and abdominal cramps position
Nausea, vomiting and headache Calf pain when the foot is dorsiflexed
42. A community health nurse makes a home visit to an Inability of the child to extend the legs fully when lying supine
elderly person living alone in a small house. Which of the Pain when the chin is pulled down to the chest
following observation would be a great concern? 48. A community health nurse makes a home visit to a child
with an infectious and communicable disease. In planning
Big mirror in a wall care for the child, the nurse must determine that the primary
Scattered and unwashed dishes in the sink goal is that the:
Shiny floors with scattered rugs
Brightly lit rooms Child will experience mild discomfort
43. The health nurse is conducting health teaching about Child will experience only minor complications
“safe” sex to a group of high school students. Which of the Child will not spread the infection to others
following statement about the use of condoms should the Public health department will be notified
nurse avoid making? 49. The mother brings her daughter to the health care clinic.
The child was diagnosed with conjunctivitis. The nurse
“Condoms should be used because they can prevent infection provides health teaching to the mother about the proper
and because they may prevent pregnancy” care of her daughter while at home. Which statement by the
“Condoms should be used even if you have recently tested mother indicates a need for additional information?
negative for HIV”
“Condoms should be used every time you have sex because “I do not need to be concerned about the spreading of this
condoms prevent all forms of sexually transmitted diseases” infection to others in my family”
“Condoms should be used every time you have sex even if “I should apply warm compresses before instilling antibiotic
you are taking the pill because condoms can prevent the drops if purulent discharge is present in my daughter’s eye”
spread of HIV and gonorrhea” “I can use an ophthalmic analgesic ointment at nighttime if I
have eye discomfort”
“I should perform a saline eye irrigation before instilling, the 10. D. The recommended position to administer rectal
antibiotic drops into my daughter’s eye if purulent discharge medications to children is side-lying with the upper leg
is present” flexed. This position allows the nurse to safely and
50. A community health nurse is caring for a group of flood effectively administer the medication while promoting
victims in Marikina area. In planning for the potential needs comfort for the child.
of this group, which is the most immediate concern? 11. C. For a 12month-old child, 110 apical pulse rate is
normal and therefore it is safe to give the digoxin. A
Finding affordable housing for the group toddler’s normal pulse rate is slightly lower than an
Peer support through structured groups infant’s (120).
Setting up a 24-hour crisis center and hotline 12. B. Chest physiotherapy treatments are scheduled
Meeting the basic needs to ensure that adequate food, between meals to prevent aspiration of stomach
shelter and clothing are available contents, because the child is placed in a variety of
positions during the treatment process.
Answers and Rationales 13. C. It is common misconception that breastfeeding may
1. C. Nothing must be placed in the mouth of a toddler prevent pregnancy.
who just undergone a cleft palate repair until the 14. B. The “security blanket” is an important transitional
suture line has completely healed. It is the nurse’s object for the toddler. It provides a feeling of comfort
responsibility to inform the parent of the client. Spoon, and safety when the maternal figure is not present or
forks, straws, and tongue blades are other when in a new situation for which the toddler was not
unacceptable items to place in the mouth of a toddler prepared. Virtually any object (stuffed animal, doll,
who just undergone cleft palate repair. The general book etc) can become a security blanket for the
principle of care is that nothing should enter the toddler.
mouth until the suture line has completely healed. 15. D. Toddlers need to meet the developmental
2. D. The pancreatic capsules contain pancreatic enzyme milestone of autonomy versus shame and doubt. In
that should be administered in a cold, not a hot, order to accomplish this, the toddler must be able to
medium (example: chilled applesauce versus hot explore and manipulate the environment.
chocolate) to maintain the medication’s integrity. 16. D. The introduction of a baby into a family with one or
3. B. When oral iron preparations are given correctly, the more children challenges parent to promote
stools normally turn dark green or black. Parents of acceptance of the baby by siblings. The parent’s
children receiving this medication should be advised attitudes toward the arrival of the baby can set the
that this side effect indicates the medication is being stage for the other children’s reaction. Spending time
absorbed and is working well. with the older siblings alone will also reassure them of
4. C. Reviewing the number of prescription refills the their place in the family, even though the older
child has required over the last 6 months would be the children will have to eventually assume new positions
best indicator of how well controlled and thus how within the family hierarchy.
effective the child’s asthma treatment is. Breakthrough 17. D. The autonomous toddler would be frustrated by
wheezing, shortness of breath, and upper respiratory being confined to be. The pounding board and hammer
infections would require that the child take additional is developmentally appropriate and an excellent way
medication. This would be reflected in the number of for the toddler to release frustration.
prescription refills. 18. D. It is the developmental task of an 18-month-old
5. D. Tetracycline may cause a phototoxic reaction. toddler to explore and learn about the environment.
6. D. The normal heart rate of an infant is 120-160 beats The respiratory complications associated with cystic
per minute. fibrosis (which are present in almost all children with
7. C. Both gentamicin and chemotherapeutic agents can cystic fibrosis) could prevent this development task
cause renal impairment and acute renal failure; thus from occurring.
baseline renal function must be evaluated before 19. C. The best diversion for a hospitalized child aged 2-3
initiating either medication. years old would be anything that makes noise or
8. C. In selecting the correct needle to administer an IM makes a mess; xylophone which certainly makes noise
injection to a preschooler, the nurse should always or music would be the best choice.
look at the child and use judgment in evaluating 20. B. An 11-month-old child stands alone and can walk
muscle mass and amount of subcutaneous fat. In this holding onto people or objects. Therefore the
case, in the absence of further data, the nurse would installation of a gate at the top and bottom of any
be most correct in selecting a needle gauge and length stairs in the house is crucial for the child’s safety.
appropriate for the “average’ preschool child. A 21. D. In second grade a child needs to form a close
medium-gauge needle (21G) that is 1 inch long would relationships with peers.
be most appropriate. 22. B. A 5-year-old is highly concerned with body
9. C. The purpose of the salicylate therapy is to relieve integrity. The preschool-age child normally asks many
the pain associated with the migratory polyarthritis questions and in a situation such as this, could be
accompanying the rheumatic fever. Playing mini piano expected to ask even more.
would require movement of the child’s joints and 23. C. A 9-year-old enjoys working and feeling a sense of
would provide the nurse with a means of evaluating accomplishment. The school-age child also enjoys
the child’s level of pain. “showing off,” and doing something with the nurse on
the pediatric unit would allow this. This activity also
provides the school-age child a needed opportunity to 39. C. This is appropriate foods that are high in iron and
interact with others in the absence of school and calcium but would not affect lactose intolerance.
personal friends. 40. D. Intervention is needed when the woman thinks that
24. B. Adolescents do feel indestructible, and this is she needs to wait only 6 months after being free of TB
reflected in many risk-taking behaviors. before she can get pregnant. She needs to wait 1.5-
25. D. An 8-month-old infant both recognizes and is fearful 2years after she is declared to be free of TB before she
of strangers. This developmental milestone is known should attempt pregnancy.
as “stranger anxiety”. 41. C. Salmonella organisms cause lower GI symptoms
26. A. The basal body temperature (BBT) is the lowest 42. C. It is a safety hazard to have shiny floors and
body temperature of a healthy person that is taken scattered rugs because they can cause falls and rugs
immediately after waking and before getting out of should be removed.
bed. The BBT usually varies from 36.2 – 36.3 degree 43. C. Condoms do not prevent ALL forms of sexually
Celsius during menses and for about 5-7 days transmitted diseases.
afterward. About the time of ovulation, a slight drop 44. A. Pregnancy is one contraindication to breast-
approximately 0.05 degree Celsius in temperature may feeding. Milk secretion is inhibited and the baby’s
be seen; after ovulation, in concert with the increasing sucking may stimulate uterine contractions.
progesterone levels of the early luteal phase, the BBT 45. C. Perianal itching is the child’s chief complaint
rises 0.2-0.4 degree Celsius. This elevation remains associated with the diagnosis of pinworms. The itching,
until 2-3 days before menstruation, or if pregnancy has in this instance, is often described as being “intense” in
occurred. nature. Pinworms infestation usually occurs because
27. C. The woman must understand that, although the the child is in the anus-to-mouth stage of development
“fertile” period is approximately midcycle, hormonal (child uses the toilet, does not wash hands, places
variations do occur and can result in early or late hands and pinworm eggs in mouth). Teaching the child
ovulations. To be effective, the diaphragm should be hand washing before eating and after using the toilet
inserted before every intercourse. can assist in breaking the cycle.
28. D. Although instructions vary among birth centers, 46. C. Severe itching of the scalp is the classic sign and
primigravidas should seek care when regular symptom of head lice in a child. In turn, this would
contractions are felt about 5 minutes apart, becoming lead to the nursing diagnosis of “altered comfort”.
longer and stronger. 47. C. Kernig’s sign is the inability of the child to extend
29. B. Prolactin suppresses estrogen, which is needed to the legs fully when lying supine. This sign is frequently
stimulate vaginal lubrication during arousal. present in bacterial meningitis. Nuchal rigidity is also
30. B. A wood surface provides the least chance for present in bacterial meningitis and occurs when pain
organisms to be present. prevents the child from touching the chin to the chest.
31. A. Preventing infection in the infant with eczema is the 48. C. The primary goal is to prevent the spread of the
nurse’s most important goal. The infant with eczema is disease to others. The child should experience no
at high risk for infection due to numerous breaks in the complication. Although the health department may
skin’s integrity. Intact skin is always the infant’s first need to be notified at some point, it is no the primary
line of defense against infection. goal. It is also important to prevent discomfort as
32. B. A skin rash could indicate a concurrent infectious much as possible.
disease process in the infant. The scheduled 49. A. Conjunctivitis is highly contagious. Antibiotic drops
immunizations should be withheld until the status of are usually administered four times a day. When
the infant’s health can be determined. Fevers above purulent discharge is present, saline eye irrigations or
38.5 degrees Celsius, alteration in skin integrity, and eye applications of warm compresses may be
infectious-appearing secretions are indications to necessary before instilling the medication. Ophthalmic
withhold immunizations. analgesic ointment or drops may be instilled, especially
33. C. Continuity is essential to promote active immunity at bedtime, because discomfort becomes more
and give hepatitis B lifelong prophylaxis. Optimally, the noticeable when the eyelids are closed.
third vaccination is given 6 months after the first. 50. D. The question asks about the immediate concern.
34. B. Because of the high incidence of breast cancer, all The ABCs of community health care are always
women are considered to be at risk regardless of attending to people’s basic needs of food, shelter, and
health history. clothing
35. B. Handwashing is the best way to prevent the spread
of infection. PNLE III Nursing Practice
36. A. The purpose of the visit takes priority. The scope of this Nursing Test III is parallel to the NP3 NLE
37. D. Frequent douching interferes with the natural Coverage:
protective barriers in the vagina that resist yeast Medical Surgical Nursing
infection and should be avoided.
38. A. Completion for the recommended schedule of infant 1. The nurse is going to replace the Pleur-O-Vac attached to
immunizations does not require a large number of the client with a small, persistent left upper lobe
immunizations, but it also provides protection against pneumothorax with a Heimlich Flutter Valve. Which of the
multiple pediatric communicable and infectious following is the best rationale for this?
diseases.
Promote air and pleural drainage
Prevent kinking of the tube following nursing measure will ease the insertion to the
Eliminate the need for a dressing tube?
Eliminate the need for a water-seal drainage
2. The client with acute pancreatitis and fluid volume deficit Positioning the client in Semi-Fowler’s position
is transferred from the ward to the ICU. Which of the Administering a sedative to reduce anxiety
following will alert the nurse? Chilling the tube before insertion
Warming the tube before insertion
Decreased pain in the fetal position 9. The physician ordered a low-sodium diet to the client.
Urine output of 35mL/hr Which of the following food will the nurse avoid to give to
CVP of 12 mmHg the client?
Cardiac output of 5L/min
3. The nurse in the morning shift is making rounds in the Orange juice.
ward. The nurse enters the client’s room and found the client Whole milk.
in discomfort condition. The client complains of stiffness in Ginger ale.
the joints. To reduce the early morning stiffness of the joints Black coffee.
of the client,the nurse can encourage the client to: 10. Mr. Bean, a 70-year-old client is admitted in the hospital
for almost one month. The nurse understands that prolonged
Sleep with a hot pad immobilization could lead to decubitus ulcers. Which of the
Take to aspirins before arising, and wait 15 minutes before following would be the least appropriate nursing intervention
attempting locomotion in the prevention of decubitus?
Take a hot tub bath or shower in the morning
Put joints through passive ROM before trying to move them Giving backrubs with alcohol
actively Use of a bed cradle
4. The nurse is planning of care to a client with peptic ulcer Frequent assessment of the skin
disease. To avoid the worsening condition of the client, the Encouraging a high-protein diet
nurse should carefully plan the diet of the client. Which of 11. The physician prescribed digoxin 0.125 mg PO qd to a
the following will be included in the diet regime of the client? client and instructed the nurse that the client is on high-
potassium diet. High potassium foods are recommended in
Eating mainly bland food and milk or dairy products the diet of a client taking digitalis preparations because a low
Reducing intake of high-fiber foods serum potassium has which of the following effects?
Eating small, frequent meals and a bedtime snack
Eliminating intake of alcohol and coffee Potentiates the action of digoxin
5. The physician has given instruction to the nurse that the Promotes calcium retention
client can be ambulated on crutches, with no weight bearing Promotes sodium excretion
on the affected limb. The nurse is aware that the appropriate Puts the client at risk for digitalis toxicity
crutch gait for the nurse to teach the client would be: 12. The nurse is caring for a client who is transferred from the
operating room for pneumonectomy. The nurse knows that
Tripod gait immediately following pneumonectomy; the client should be
Two-point gait in what position?
Four-point gait
Three-point gait Supine on the unaffected side
6. The client is transferred to the nursing care unit from the Low-Fowler’s on the back
operating room after a transurethral resection of the Semi-Fowler’s on the affected side
prostate. The client is complaining of pain in the abdomen Semi-Fowler’s on the unaffected side
area. The nurse suspects of bladder spasms, which of the 13. A client is placed on digoxin, high potassium foods are
following is the best nursing action to minimize the pain felt recommended in the diet of the client. Which of the
by the client? following foods willthe nurse give to the client?

Advising the client not to urinate around catheter Whole grain cereal, orange juice, and apricots
Intermittent catheter irrigation with saline Turkey, green bean, and Italian bread
Giving prescribed narcotics every 4 hour Cottage cheese, cooked broccoli, and roast beef
Repositioning catheter to relieve pressure Fish, green beans and cherry pie
7. A client is diagnosed with peptic ulcer. The nurse caring for 14. The nurse is assigned to care to a client who undergone
the client expects the physician to order which diet? thyroidectomy. What nursing intervention is important
during the immediate postoperative period following a
NPO thyroidectomy?
Small feedings of bland food
A regular diet given frequently in small amounts Assess extremities for weakness and flaccidity
Frequent feedings of clear liquids Support the head and neck during position changes
8. The nurse is going to insert a Miller-Abbott tube to the Position the client in high Fowler’s
client. Before insertion of the tube, the balloon is tested for Medicate for restlessness and anxiety
patency and capacity and then deflated. Which of the
15. What would be the recommended diet the nurse will the nurse, his temperature is 38.1 ºC. The physician ordered
implement to a client with burns of the head, face, neck and Morphine sulfate via patient-controlled analgesia (PCA), and
anterior chest? oxygen at 4L/min. A priority nursing diagnosis to Mr. Stewart
is risk for infection. A nursing intervention to assist in
Serve a high-protein, high-carbohydrate diet preventing infection is:
Encourage full liquid diet
Serve a high-fat diet, high-fiber diet Using standard precautions and medical asepsis
Monitor intake to prevent weight gain Enforcing a “no visitors” rule
16. A client with multiple fractures of both lower extremities Using moist heat on painful joints
is admitted for 3 days ago and is on skeletal traction. The Monitoring a vital signs every 2 hour
client is complaining of having difficulty in bowel movement. 22. Mrs. Maupin is a professor in a prestigious university for
Which of the following would be the most appropriate 30 years. After lecture, she experience blurring of vision and
nursing intervention? tiredness. Mrs. Maupin is brought to the emergency
department. On assessment, the nurse notes that the blood
Administer an enema pressure of the client is 139/90. Mrs. Maupin has been
Perform range-of-motion exercise to all extremities diagnosed with essential hypertension and placed on
Ensure maximum fluid intake (3000ml/day) medication to control her BP. Which potential nursing
Put the client on the bedpan every 2 hours diagnosis will be a priority for discharge teaching?
17. John is diagnosed with Addison’s disease and admitted in
the hospital. What would be the appropriate nursing care for Sleep Pattern disturbance
John? Impaired physical mobility
Noncompliance
Reducing physical and emotional stress Fluid volume excess
Providing a low-sodium diet 23. Following a needle biopsy of the kidney, which
Restricting fluids to 1500ml/day assessment is an indication that the client is bleeding?
Administering insulin-replacement therapy
18. Mr. Smith is scheduled for an above-the-knee Slow, irregular pulse
amputation. After the surgery he was transferred to the Dull, abdominal discomfort
nursing care unit. The nurse assigned to him knows that 72 Urinary frequency
hours after the procedure the client should be positioned Throbbing headache
properly to prevent contractures. Which of the following is 24. A client with acute bronchitis is admitted in the hospital.
the best position to the client? The nurse assigned to the client is making a plan of care
regarding expectoration of thick sputum. Which nursing
Side-lying, alternating left and right sides action is most effective?
Sitting in a reclining chair twice a day
Lying on abdomen several times daily Place the client in a lateral position every 2 hour
Supine with stump elevated at least 30 degrees Splint the patient’s chest with pillows when coughing
19. A client is scheduled to have an inguinal herniorraphy in Use humified oxygen
the outpatient surgical department. The nurse is providing Offer fluids at regular intervals
health teaching about post surgical care to the client. Which 25. The nurse is going to assess the bowel sound of the client.
of the following statement if made by the client would reflect For accurate assessment of the bowel sound, the nurse
the need for more teaching? should listen for at least:

“I should call the physician if I have a cough or cold before 5 minutes


surgery” 60 seconds
“I will be able to drive soon after surgery” 30 seconds
“I will not be able to do any heavy lifting for 3-6 weeks after 2 minutes
surgery” 26. The nurse encourages the client to wear compression
“I should support my incision if I have to cough or turn” stockings. What is the rationale behind in using compression
20. Ms Jones is brought to the emergency room and is stockings?
complaining of muscle spasms, numbness, tremors and
weakness in the arms and legs. The client was diagnosed with Compression stockings promote venous return
multiple sclerosis. The nurse assigned to Ms. Jones is aware Compression stockings divert blood to major vessels
that she has to prevent fatigue to the client to alleviate the Compression stockings decreases workload on the heart
discomfort. Which of the following teaching is necessary to Compression stockings improve arterial circulation
prevent fatigue? 27. Mr. Whitman is a stroke client and is having difficulty in
swallowing. Which is the best nursing intervention is most
Avoid extremes in temperature likely to assist the client?
Install safety devices in the home
Attend support group meetings Placing food in the unaffected side of the mouth
Avoid physical exercise Increasing fiber in the diet
21. Mr. Stewart is in sickle cell crisis and complaining pain in Asking the patient to speak slowly
the joints and difficulty of breathing. On the assessment of Increasing fluid intake
28. Following nephrectomy, the nurse closely monitors the Give a stat dose of Sucralfate (Carafate)
urinary output of the client. Which assessment finding is an 35. After a right lower lobectomy on a 55-year-old client,
early indicator of fluid retention in the postoperative period? which action should the nurse initiate when the client is
transferred from the post anesthesia care unit?
Periorbital edema
Increased specific gravity of urine Notify the family to report the client’s condition
A urinary output of 50mL/hr Immediately administer the narcotic as ordered
Daily weight gain of 2 lb or more Keep client on right side supported by pillows
29. A nurse is completing an assessment to a client with Encourage coughing and deep breathing every 2 hours
cirrhosis. Which of the following nursing assessment is 36. The nurse is providing a discharge instruction about the
important to notify the physician? prevention of urinary stasis to a client with frequent bladder
infection. Which of the following will the nurse include in the
Expanding ecchymosis instruction?
Ascites and serum albumin of 3.2 g/dl
Slurred speech Drink 3-4 quarts of fluid every day
Hematocrit of 37% and hemoglobin of 12g/dl Empty the bladder every 2-4 hours while awake
30. Mr. Park is 32-year-old, a badminton player and has a Encourage the use of coffee, tea, and colas for their diuretic
type 1 diabetes mellitus. After the game, the client complains effect
of becoming diaphoretic and light-headedness. The client Teach Kegel exercises to control bladder flow
asks the nurse how to avoid this reaction. The nurse will 37. A male client visits the clinic for check-up. The client tells
recommend to: the nurse that there is a yellow discharge from his penis. He
also experiences a burning sensation when urinating. The
Allow plenty of time after the insulin injection and before nurse is suspecting of gonorrhea. What teaching is necessary
beginning the match for this client?
Eat a carbohydrate snack before and during the badminton
match Sex partner of 3 months ago must be treated
Drink plenty of fluids before, during, and after bed time Women with gonorrhea are symptomatic
Take insulin just before starting the badminton match Use a condom for sexual activity
31. A client is rushed to the emergency room due to serious Sex partner needs to be evaluated
vehicle accident. The nurse is suspecting of head injury. 38. A client with AIDS is admitted in the hospital. He is
Which of the following assessment findings would the nurse receiving intravenous therapy. While the nurse is assessing
report to the physician? the IV site, the client becomes confused and restless and the
intravenous catheter becomes disconnected and minimal
CVP of 5mmHa amount of the client’s blood spills onto the floor. Which
Glasgow Coma Scale score of 13 action will the nurse take to remove the blood spill?
Polyuria and dilute urinary output
Insomnia Promptly clean with a 1:10 solution of household bleach and
32. Mrs. Moore, 62-year-old, with diabetes is in the water
emergency department. She stepped on a sharp sea shells Promptly clean up the blood spill with full-strength
while walking barefoot along the beach. Mrs. Moore did not antimicrobial cleaning solution
notice that the object pierced the skin until later that Immediately mop the floor with boiling water
evening. What problem does the client most probably have? Allow the blood to dry before cleaning to decrease the
possibility of cross-contamination
Nephropathy 39. Before surgery, the physician ordered pentobarbital
Macroangiopathy sodium (Nembutal) for the client to sleep. The night before
Carpal tunnel syndrome the scheduled surgery, the nurse gave the pre-medication.
Peripheral neuropathy One hour later the client is still unable to sleep. The nurse
33. A client with gangrenous foot has undergone a below- review the client’s chart and note the physician’s prescription
knee amputation. The nurse in the nursing care unit knows with an order to repeat. What should the nurse do next?
that the priority nursing intervention in the immediate post
operative care of this client is: Rub the client’s back until relaxed
Prepare a glass of warm milk
Elevate the stump on a pillow for the first 24 hours Give the second dose of pentobarbital sodium
Encourage use of trapeze Explore the client’s feelings about surgery
Position the client prone periodically 40. The nurse on the night shift is making rounds in the
Apply a cone-shaped dressing nursing care unit. The nurse is about to enter to the client’s
34. A client with a diagnosis of gastric ulcer is complaining of room when a ventilator alarm sounds, what is the first action
syncope and vertigo. What would be the initial nursing the nurse should do?
intervention by the nurse?
Assess the lung sounds
Monitor the client’s vital signs Suction the client right away
Keep the client on bed rest Look at the client
Keep the patient on bed rest Turn and position the client
41. What effective precautions should the nurse use to
control the transmission of methicillin-resistant Call the physician
Staphylococcus aureus (MRSA)? Give a prn pain medication
Clarify if the client is on a new medication
Use gloves and handwashing before and after client contact Use gown and gloves while assessing the lesions
Do nasal cultures on healthcare providers 48. A client is admitted and has been diagnosed with
Place the client on total isolation bacterial (meningococcal) meningitis. The infection control
Use mask and gown during care of the MRSA client registered nurse visits the staff nurse caring to the client.
42. The postoperative gastrectomy client is scheduled for What statement made by the nurse reflects an understanding
discharge. The client asks the nurse, “When I will be allowed of the management of this client?
to eat three meals a day like the rest of my family?”. The
appropriate nursing response is: speech pattern may be altered
Respiratory isolation is necessary for 24 hours after
“You will probably have to eat six meals a day for the rest of antibiotics are started
your life.” Perform skin culture on the macular popular rash
“Eating six meals a day can be a bother, can’t it?” Expect abnormal general muscle contractions
“Some clients can tolerate three meals a day by the time they 49. A 18-year-old male client had sustained a head injury
leave the hospital. Maybe it will be a little longer for you.” from a motorbike accident. It is uncertain whether the client
“ It varies from client to client, but generally in 6-12 months may have minimal but permanent disability. The family is
most clients can return to their previous meal patterns” concerned regarding the client’s difficulty accepting the
43. A male client with cirrhosis is complaining of belly pain, possibility of long term effects. Which nursing diagnosis is
itchiness and his breasts are getting larger and also the best for this situation?
abdomen. The client is so upset because of the discomfort
and asks the nurse why his breast and abdomen are getting Nutrition, less than body requirements
larger. Which of the following is the appropriate nursing Injury, potential for sensory-perceptual alterations
response? Impaired mobility, related to muscle weakness
Anticipatory grieving, due to the loss of independence
“How much of a difference have you noticed” 50. A client with AIDS is scheduled for discharge. The client
“It’s part of the swelling your body is experiencing” tells the nurse that one of his hobbies at home is gardening.
“It’s probably because you have been less physically active” What will be the discharge instruction of the nurse to the
“Your liver is not destroying estrogen hormones that all men client knowing that the client is prone to toxoplasmosis?
produce”
44. A client is diagnosed with detached retina and scheduled Wash all vegetables before cooking
for surgery. Preoperative teaching of the nurse to the client Wear gloves when gardening
includes: Wear a mask when travelling to foreign countries
Avoid contact with cats and birds
No eye pain is expected postoperatively
Semi-fowler’s position will be used to reduce pressure in the Answers and Rationales
eye. 1. D. The Heimlich flutter valve has a one-way valve that
Eye patches may be used postoperatively allows air and fluid to drain. Underwater seal drainage
Return of normal vision is expected following surgery is not necessary. This can be connected to a drainage
45. A 70-year-old client is brought to the emergency bag for the patient’s mobility. The absence of a long
department with a caregiver. The client has manifestations of drainage tubing and the presence of a one-way valve
anorexia, wasting of muscles and multiple bruises. What promote effective therapy
nursing interventions would the nurse implement? 2. C. C = the normal CVP is 0-8 mmHg. This value reflects
hypervolemia. The right ventricular function of this
Talk to the client about the caregiver and support system client reflects fluid volume overload, and the physician
Complete a gastrointestinal and neurological assessment should be notified.
Check the lab data for serum albumin, hematocrit and 3. C. A hot tub bath or shower in the morning helps many
hemoglobin patients limber up and reduces the symptoms of early
Complete a police report on elder abuse morning stiffness. Cold and ice packs are used to a
46. A nurse is providing a discharge instruction to the client lesser degree, though some clients state that cold
about the self-catheterization at home. Which of the decreases localized pain, particularly during acute
following instructions would the nurse include? attacks.
4. D. These substances stimulate the production of
Wash the catheter with soap and water after each use hydrochloric acid, which is detrimental in peptic ulcer
Lubricate the catheter with Vaseline disease.
Perform the Valsalva maneuver to promote insertion 5. D. The three-point gait is appropriate when weight
Replace the catheter with a new one every 24 hour bearing is not allowed on the affected limb. The swing-
47. The nurse in the nursing care unit is assigned to care to a to and swing-through crutch gaits may also be used
client who is Immunocompromised. The client tells the nurse when only one leg can be used for weight bearing
that his chest is painful and the blisters are itchy. What would 6. A. The client needs to be told before surgery that the
be the nursing intervention to this client? catheter causes the urge to void. Attempts to void
around the catheter cause the bladder muscles to 25. D. Physical assessment guidelines recommend listening
contract and result in painful spasms. for atleast 2 minutes in each quadrant (and up to 5
7. B. Bland feedings should be given in small amounts on minutes, not at least 5 minutes).
a frequent basis to neutralize the hydrochloric acid and 26. A. Compression stockings promote venous return and
to prevent overload prevent peripheral pooling.
8. C. Chilling the tube before insertion assists in relieving 27. A. Placing food in the unaffected side of the mouth
some of the nasal discomfort. Water-soluble lubricants assists in the swallowing process because the client has
along with viscous lidocaine (Xylocaine) may also be sensation on that side and will have more control over
used. It is usually only lightly lubricated before the swallowing process.
insertion 28. D. Daily weights are taken following nephrectomy.
9. B. Whole milk should be avoided to include in the Daily increases of 2 lb or more are indicative of fluid
client’s diet because it has 120 mg of sodium in 8 0z of retention and should be reported to the physician.
milk. Intake and output records may also reflect this
10. A. Alcohol is extremely drying and contributes to skin imbalance.
break down. An emollient lotion should be used. 29. A. Clients with cirrhosis have already coagulation due
11. D. Potassium influences the excitability of nerves and to thrombocytopenia and vitamin K deficiency. This
muscles. When potassium is low and the client is on could be a sign of bleeding
digoxin, the risk of digoxin toxicity is increased. 30. B. Exercise enhances glucose uptake, and the client is
12. C. This position allows maximum expansion, at risk for an insulin reaction. Snacks with
ventilation, and perfusion of the remaining lung. carbohydrates will help.
13. A. These foods are high in potassium 31. C. These are symptoms of diabetes insipidus. The
14. B. Stress on the suture line should be avoided. Prevent patient can become hypovolemic and vasopressin may
flexion or hyperextension of the neck, and provide a reverse the Polyuria.
small pillow under thehead and neck. Neck muscles 32. D. Peripheral neuropathy refers to nerve damage of
have been affected during a thyroidectomy, support the hands and feet. The client did not notice that the
essential for comfort and incisional support. object pierced the skin.
15. A. A positive nitrogen balance is important for meeting 33. A. The elevation of the stump on a pillow for the first
metabolic needs, tissue repair, and resistance to 24 hours decreases edema and increases venous
infection. Caloric goals may be as high as 5000 calories return.
per day. 34. B. The priority is to maintain client’s safety. With
16. C. The best early intervention would be to increase syncope and vertigo, the client is at high risk for falling.
fluid intake, because constipation is common when 35. D. Coughing and deep breathing are essential for re-
activity is decreased or usual routines have been expansion of the lung
interrupted. 36. B. Avoiding stasis of urine by emptying the bladder
17. A. Because the client’s ability is to react to stress is every 2-4 hours will prevent overdistention of the
decreased, maintaining a quiet environment becomes bladder and future urinary tract infections.
A nursing priority. Dehydration is a common problem 37. D. If infected, the sex partner must be evaluated and
in Addison’s disease, so close observation of the treated
client’s hydration level is crucial. To promote optimal 38. A. A 1:10 solution of household bleach and water is
hydration and sodium intake, fluid intake is increased, recommended by the Centers for Disease Control and
particularly fluid containing electrolytes, such as Prevention to kill the human immunodeficiency virus
broths, carbonated beverages, and juices. (HIV).
18. C. At about 48-72 hours, the client must be turned 39. D. Given the data, presurgical anxiety is suspected. The
onto the abdomen to prevent flexion contractures. client needs an opportunity to talk about concerns
19. B. The client should not drive for 2 weeks after surgery related to surgery before further actions (which may
to avoid stress on the incision. This reflects a need for mask the anxiety).
additional teaching. 40. C. A quick look at the client can help identify the type
20. A. Extremes in heat and cold will exacerbate and cause of the ventilator alarm. Disconnection of the
symptoms. Heat delays transmission of impulses and tube from the ventilator, bronchospasm, and anxiety
increases fatigue. are some of the obvious reasons that could trigger an
21. A. Vigilant implementation of standard precautions alarm.
and medical asepsis is an effective means of 41. A. Contact isolation has been advised by the Centers
preventing infection for Disease Control and Prevention (CDC) to control
22. C. Noncompliance is a major problem in the transmission of MRSA, which includes gloves and
management of chronic disease. In hypertension, the handwashing.
client often does not feel ill and thus does not see a 42. D. In response to the question of the client, the nurse
need to follow a treatment regimen. needs to provide brief, accurate information. Some
23. B. An accumulation of blood from the kidney into the clients who have had gastrectomies are able to
abdomen would manifest itself with these symptoms tolerate three meals a day before discharge from the
24. D. Fluids liquefy secretions and therefore make it hospital. However, for the majority of clients, it takes
easier to expectorate 6-12 months before their surgically reduced stomach
has stretched enough to accommodate a larger meal.
43. A. This allows the client to elaborate his concern and 4. A client is diagnosed with a brain tumor in the occipital
provides the nurse a baseline of assessment lobe. Which of the following will the client most likely
44. C. Use of eye patches may be continued experience?
postoperatively, depending on surgeon preference.
This is done to achieve >90% success rate of the Visual hallucinations.
surgery. Receptive aphasia.
45. B. Assessment and more data collection are needed. Hemiparesis.
The client may have gastrointestinal or neurological Personality changes.
problems that account for the symptoms. The anorexia 5. A client with Addison’s disease has a blood pressure of
could result from medications, poor dentition, or 65/60. The nurse understands that decreased blood pressure
indigestion, the bruises may be attributed to ataxia, of the client with Addison’s disease involves a disturbance in
frequent falls, vertigo, or medication. the production of:
46. A. The catheter should be washed with soap and water
after withdrawal and placed in a clean container. It can Androgens
be reused until it is too hard or too soft for insertion. Glucocorticoids
Self-care, prevention of complications, and cost- Mineralocorticoids
effectiveness are important in home management. Estrogen
47. D. The client may have herpes zoster (shingles), a viral 6. The nurse is planning to teach the client about a
infection. The nurse should use standard precautions spontaneous pneumothorax. The nurse would base the
in assessing the lesions. Immunocompromised clients teaching on the understanding that:
are at risk for infection.
48. B. After a minimum of 24 hours of IV antibiotics, the Inspired air will move from the lung into the pleural space.
client is no longer considered communicable. There is greater negative pressure within the chest cavity.
Evaluation of the nurse’s knowledge is needed for safe The heart and great vessels shift to the affected side.
care and continuity of care. The other lung will collapse if not treated immediately.
49. D. Stem of the question supports this choice by stating 7. During an assessment, the nurse recognizes that the client
that the client has difficulty accepting the potential has an increased risk for developing cancer of the tongue.
disability. Which of the following health history will be a concern?
50. B. Toxoplasmosis is an opportunistic infection and a
parasite of birds and mammals. The oocysts remain Heavy consumption of alcohol.
infectious in moist soil for about 1 year. Frequent gum chewing.
Nail biting.
PNLE IV Nursing Practice Poor dental habits.
The scope of this Nursing Test IV is parallel to the NP4 NLE 8. The client in the orthopedic unit asks the nurse the reason
Coverage: behind why compact bone is stronger than cancellous bone.
Medical Surgical Nursing Which of the following is the correct response of the nurse?

1. Following spinal injury, the nurse should encourage the Compact bone is stronger than cancellous bone because of its
client to drink fluids to avoid: greater size.
Compact bone is stronger than cancellous bone because of its
Urinary tract infection. greater weight.
Fluid and electrolyte imbalance. Compact bone is stronger than cancellous bone because of its
Dehydration. greater volume.
Skin breakdown. Compact bone is stronger than cancellous bone because of its
2. The client is transferred from the operating room to greater density.
recovery room after an open-heart surgery. The nurse 9. The nurse is reviewing the laboratory results of the client.
assigned is taking the vital signs of the client. The nurse In reviewing the results of the RBC count, the nurse
notified the physician when the temperature of the client understands that the higher the red blood cell count, the :
rises to 38.8 ºC or 102 ºF because elevated temperatures:
Greater the blood viscosity.
May be a forerunner of hemorrhage. Higher the blood pH.
Are related to diaphoresis and possible chilling. Less it contributes to immunity.
May indicate cerebral edema. Lower the hematocrit.
Increase the cardiac output. 10. The physician advised the client with Hemiparesis to use a
3. After radiation therapy for cancer of the prostate, the cane. The client asks the nurse why cane will be needed. The
client experienced irritation in the bladder. Which of the nurse explains to the client that cane is advised specifically
following sign of bladder irritability is correct? to:

Hematuria Aid in controlling involuntary muscle movements.


Dysuria Relieve pressure on weight-bearing joints.
Polyuria Maintain balance and improve stability.
Dribbling Prevent further injury to weakened muscles.
11. The nurse is conducting a discharge teaching regarding 17. A client with AIDS develops bacterial pneumonia is
the prevention of further problems to a client who admitted in the emergency department. The client’s arterial
undergone surgery for carpal tunnel syndrome of the right blood gases is drawn and the result is PaO2 80mmHg. then
hand. Which of the following instruction will the nurse arterial blood gases are drawn again and the level is reduced
includes? from 80 mmHg to 65 mmHg. The nurse should;

Learn to type using your left hand only. Have arterial blood gases performed again to check for
Avoid typing in a long period of time. accuracy.
Avoid carrying heavy things using the right hand. Increase the oxygen flow rate.
Do manual stretching exercise during breaks. Notify the physician.
12. A female client is admitted because of recurrent urinary Decrease the tension of oxygen in the plasma.
tract infections. The client asks the nurse why she is prone to 18. An 18-year-old college student is brought to the
this disease. The nurse states that the client is most emergency department due to serious motor vehicle
susceptible because of: accident. Right above-knee-amputation is done. Upon
awakening from surgery the client tells the nurse, “What
Continuity of the mucous membrane. happened to me? I cannot remember anything?” Which of
Inadequate fluid intake. the following would be the appropriate initial nursing
The length of the urethra. response?
Poor hygienic practices.
13. A 55-year-old client is admitted with chest pain that “You sound concerned; You’ll probably remember more as
radiates to the neck, jaw and shoulders that occurs at rest, you wake up.”
with high body temperature, weak with generalized sweating “Tell me what you think happened.”
and with decreased blood pressure. A myocardial infarction is “You were in a car accident this morning.”
diagnosed. The nurse knows that the most accurate “An amputation of your right leg was necessary because of an
explanation for one of these presenting adaptations is: accident.”
19. A 38-year-old client with severe hypertension is
Catecholamines released at the site of the infarction causes hospitalized. The physician prescribed a Captopril (Capoten)
intermittent localized pain. and Alprazolam (Xanax) for treatment. The client tells the
Parasympathetic reflexes from the infarcted myocardium nurse that there is something wrong with the medication and
causes diaphoresis. nursing care. The nurse recognizes this behavior is probably a
Constriction of central and peripheral blood vessels causes a manifestation of the client’s:
decrease in blood pressure.
Inflammation in the myocardium causes a rise in the systemic Reaction to hypertensive medications.
body temperature. Denial of illness.
14. Following an amputation of a lower limb to a male client, Response to cerebral anoxia.
the nurse provides an instruction on how to prevent a hip Fear of the health problem.
flexion contracture. The nurse should instruct the client to:. 20. Before discharge, the nurse scheduled the client who had
a colostomy for colorectal cancer for discharge instruction
Perform quadriceps muscle setting exercises twice a day. about resuming activities. The nurse should plan to help the
Sit in a chair for 30 minutes three times a day. client understands that:
Lie on the abdomen 30 minutes every four hours.
Turn from side to side every 2 hours. After surgery, changes in activities must be made to
15. The physician scheduled the client with rheumatoid accommodate for the physiologic changes caused by the
arthritis for the injection of hydrocortisone into the knee operation.
joint. The client asks the nurse why there is a need for this Most sports activities, except for swimming, can be resumed
injection. The nurse explains that the most important reason based on the client’s overall physical condition.
for doing this is to: With counseling and medical guidance, a near normal
lifestyle, including complete sexual function is possible.
Lubricate the joint. Activities of daily living should be resumed as quickly as
Prevent ankylosis of the joint. possible to avoid depression and further dependency.
Reduce inflammation. 21. A client is scheduled for bariatric surgery. Preoperative
Provide physiotherapy. teaching is done. Which of the following statement would
16. The nurse is assigned to care for a 57-year-old female alert the nurse that further teaching to the client is
client who had a cataract surgery an hour ago. The nurse necessary?
should:
“I will be limiting my intake to 600 to 800 calories a day once
Advise the client to refrain from vigorous brushing of teeth I start eating again.”
and hair. “I’m going to have a figure like a model in about a year.”
Instruct the client to avoid driving for 2 weeks. “I need to eat more high-protein foods.”
Encourage eye exercises to strengthen the ocular “I will be going to be out of bed and sitting in a chair the first
musculature. day after surgery.”.
Teach the client coughing and deep-breathing techniques.
22. The client who had transverse colostomy asks the nurse 29. A client is receiving simvastatin (Zocor). The nurse is
about the possible effect of the surgery on future sexual aware that this medication is effective when there is
relationship. What would be the best nursing response? decrease in:

The surgery will temporarily decrease the client’s sexual The triglycerides
impulses. The INR
Sexual relationships must be curtailed for several weeks. Chest pain
The partner should be told about the surgery before any Blood pressure
sexual activity. 30. A client is taking nitroglycerine tablets, the nurse should
The client will be able to resume normal sexual relationships. teach the client the importance of:
23. A 75-year-old male client tells the nurse that his wife has
osteoporosis and asks what chances he had of getting also Increasing the number of tablets if dizziness or hypertension
osteoporosis like his wife. Which of the following is the occurs.
correct response of the nurse? Limiting the number of tablets to 4 per day.
Making certain the medication is stored in a dark container.
“This is only a problem for women.” Discontinuing the medication if a headache develops.
“You are not at risk because of your small frame.” 31. The physician prescribes Ibuprofen (Motrin) and
“You might think about having a bone density test,” hydroxychloroquine sulfate (Plaquenil) for a 58-year-old male
“Exercise is a good way to prevent this problem.” client with arthritis. The nurse provides information about
24. An older adult client with acute pain is admitted in the toxicity of the hydroxychloroquine. The nurse can determine
hospital. The nurse understands that in managing acute pain if the information is clearly understood if the client states:
of the client during the first 24 hours, the nurse should
ensure that: “I will contact the physician immediately if I develop blurred
vision.”
Ordered PRN analgesics are administered on a scheduled “I will contact the physician immediately if I develop urinary
basis. retention.”
Patient controlled analgesia is avoided in this population. “I will contact the physician immediately if I develop
Pain medication is ordered via the intramuscular route. swallowing difficulty.”
An order for meperidine (Demerol) is secured for pain relief. “I will contact the physician immediately if I develop feelings
25. A nurse is caring to an older adult with presbycusis. In of irritability.”
formulating nursing care plan for this client, the nurse should 32. The client with an acute myocardial infarction is
expect that hearing loss of the client that is caused by aging hospitalized for almost one week. The client experiences
to have: nausea and loss of appetite. The nurse caring for the client
recognizes that these symptoms may indicate the:
Overgrowth of the epithelial auditory lining.
Copious, moist cerumen. Adverse effects of spironolactone (Aldactone)
Difficulty hearing women’s voices. Adverse effects of digoxin (Lanoxin)
Tears in the tympanic membrane. Therapeutic effects of propranolol (Indiral)
26. The nurse is reviewing the client’s chart about the Therapeutic effects of furosemide (Lasix)
ordered medication. The nurse must observe for signs of 33. A client with a partial occlusion of the left common
hyperkalemia when administering: carotid artery is scheduled for discharge. The client is still
receiving Coumadin. The nurse provided a discharge
Furosemide (Lasix) instruction to the client regarding adverse effects of
Hydrochlorothiazide (HydroDIURIL) Coumadin. The nurse should tell the client to consult with the
Metolazone (Zaroxolyn) physician if:
Spironolactone (Aldactone)
27. The physician prescribed Albuterol (Proventil) to the Swelling of the ankles increases.
client with severe asthma. After the administration of the Blood appears in the urine.
medication the nurse should monitor the client for: Increased transient Ischemic attacks occur.
The ability to concentrate diminishes.
Palpitation 34. Levodopa is ordered for a client with Parkinson’s disease.
Visual disturbance Before starting the medication, the nurse should know that:
Decreased pulse rate
Lethargy Levodopa is inadequately absorbed if given with meals.
28. A client is receiving diltiazem (Cardizem). What should the Levodopa may cause the side effects of orthostatic
nurse include in a teaching plan aimed at reducing the side hypotension.
effects of this medication? Levodopa must be monitored by weekly laboratory tests.
Levodopa causes an initial euphoria followed by depression.
Take the drug with an antacid. 35. In making a diagnosis of myasthenia gravis Edrophonium
Lie down after meals. HCI (Tensilon) is used. The nurse knows that this drug will
Avoid dairy products in diet. cause a temporary increase in:
Change positions slowly.
Muscle strength
Symptoms Return of cortisone production by the adrenal glands.
Blood pressure Production of antibodies by the immune system
Consciousness Building of glycogen and protein stores in liver and muscle
36. The nurse can determine the effectiveness of Time to observe for return of increases intracranial pressure
carbamazepine (Tegretol) in the management of trigeminal 43. The nurse is assigned to care for a client with diarrhea.
neuralgia by monitoring the client’s: Excessive fluid loss is expected. The nurse is aware that fluid
deficit can most accurately be assessed by:
Seizure activity
Liver function The presence of dry skin
Cardiac output A change in body weight
Pain relief An altered general appearance
37. Administration of potassium iodide solution is ordered to A decrease in blood pressure
the client who will undergo a subtotal thyroidectomy. The 44. Which of the following is the most important electrolyte
nurse understands that this medication is given to: of intracellular fluid?

Ablate the cells of the thyroid gland that produce T4. Potassium
Decrease the total basal metabolic rate. Sodium
Decrease the size and vascularity of the thyroid. Chloride
Maintain function of the parathyroid gland. Calcium
38. A client with Addison’s disease is scheduled for discharge. 45. Which of the following client has a high risk for
Before the discharge, the physician prescribes hydrocortisone developing hyperkalemia?
and fludrocortisone. The nurse expects the hydrocortisone
to: Crohn’s disease
End-Stage renal disease
Increase amounts of angiotensin II to raise the client’s blood Cushing’s syndrome
pressure. Chronic heart failure
Control excessive loss of potassium salts. 46. The nurse is reviewing the laboratory result of the client.
Prevent hypoglycemia and permit the client to respond to The client’s serum potassium level is 5.8 mEq/L. Which of the
stress. following is the initial nursing action?
Decrease cardiac dysrhythmias and dyspnea.
39. A client with diabetes insipidus is taking Desmopressin Call the cardiac arrest team to alert them
acetate (DDAVP). To determine if the drug is effective, the Call the laboratory and repeat the test
nurse should monitor the client’s: Take the client’s vital signs and notify the physician
Obtain an ECG strip and have lidocaine available
Arterial blood pH 47. Potassium chloride, 20 mEq, is ordered and to be added
Pulse rate in the IV solution of a client in a diabetic ketoacidosis. The
Serum glucose primary reason for administering this drug is:
Intake and output
40. A client with recurrent urinary tract infections is to be Replacement of excessive losses
discharged. The client will be taking nitrofurantoin Treatment of hyperpnea
(Macrobid) 50 mg po every evening at home. The nurse Prevention of flaccid paralysis
provides discharge instructions to the client. Which of the Treatment of cardiac dysrhythmias
following instructions will be correct? 48. A female client is brought to the emergency unit. The
client is complaining of abdominal cramps. On assessment,
Strain urine for crystals and stones client is experiencing anorexia and weight is reduced. The
Increase fluid intake. physician’s diagnosis is colitis. Which of the following
Stop the drug if the urinary output increases symptoms of fluid and electrolyte imbalance should the
Maintain the exact time schedule for drug taking. nurse report immediately?
41. A client with cancer of the lung is receiving
chemotherapy. The physician orders antibiotic therapy for Skin rash, diarrhea, and diplopia
the client. The nurse understands that chemotherapy Development of tetaniy with muscles spasms
destroys rapidly growing leukocytes in the: Extreme muscle weakness and tachycardia
Nausea, vomiting, and leg and stomach cramps.
Bone marrow 49. The client is to receive an IV piggyback medication. When
Liver preparing the medication the nurse should be aware that it is
Lymph nodes very important to:
Blood
42. The physician reduced the client’s Dexamethasone Use strict sterile technique
(Decadron) dosage gradually and to continue a lower Use exactly 100mL of fluid to mix the medication
maintenance dosage. The client asks the nurse about the Change the needle just before adding the medication
change of dosage. The nurse explains to the client that the Rotate the bag after adding the medication
purpose of gradual dosage reduction is to allow: 50. The nurse is reviewing the laboratory result of the client.
An arterial blood gas report indicates the client’s pH is 7.20,
PCO2 35 mmHg and HCO3 is 19 mEq/L. The results are 18. C. This is truthful and provides basic information that
consistent with: may prompt recollection of what happened; it is a
starting point.
Metabolic acidosis 19. D. Clients adapting to illness frequently feel afraid and
Metabolic alkalosis helpless and strike out at health team members as a
Respiratory acidosis way of maintaining control or denying their fear.
Respiratory alkalosis 20. C. There are few physical restraints on activity
postoperatively, but the client may have emotional
Answers and Rationales problems resulting from the body image changes.
1. A. Clients in the early stage of spinal cord damage 21. B. Clients need to be prepared emotionally for the
experience an atonic bladder, which is characterized by body image changes that occur after bariatric surgery.
the absence of muscle tone, an enlarged capacity, no Clients generally experience excessive abdominal skin
feeling of discomfort with distention, and overflow folds after weight stabilizes, which may require a
with a large residual. This leads to urinary stasis and panniculectomy. Body image disturbance often occurs
infection. High fluid intake limits urinary stasis and in response to incorrectly estimating one’s size; it is
infection by diluting the urine and increasing urinary not uncommon for the client to still feel fat no matter
output. how much weight is lost.
2. D. The temperature of 102 ºF (38.8ºC) or greater lead 22. D. Surgery on the bowel has no direct anatomic or
to an increased metabolism and cardiac workload. physiologic effect on sexual performance. However,
3. B. Dysuria, nocturia, and urgency are all signs an the nurse should encourage verbalization.
irritable bladder after radiation therapy. 23. C. Osteoporosis is not restricted to women; it is a
4. A. The occipital lobe is involve with visual potential major health problem of all older adults;
interpretation. estimates indicate that half of all women have at least
5. C. Mineralocorticoids such as aldosterone cause the one osteoporitic fracture and the risk in men is
kidneys to retain sodium ions. With sodium, water is estimated between 13% and 25%; a bone mineral
also retained, elevating blood pressure. Absence of density measurement assesses the mass of bone per
this hormone thus causes hypotension. unit volume or how tightly the bone is packed.
6. B. As a person with a tear in the lung inhales, air 24. A. Around-the-clock administration of analgesics is
moves through that opening into the intrapleural and recommended for acute pain in the older adult
causes partial or complete collapse of the lungs. population; this help to maintain a therapeutic blood
7. A. Heavy alcohol ingestion predisposes an individual to level of pain medication.
the development of oral cancer. 25. C. Generally, female voices have a higher pitch than
8. D. The greater the density of compact bone makes it male voices; older adults with presbycusis (hearing loss
stronger than the cancellous bone. Compact bone caused by the aging process) have more difficulty
forms from cancellous bone by the addition of hearing higher-pitched sounds.
concentric rings of bones substances to the marrow 26. D. Aldactone is a potassium-sparing diuretic;
spaces of cancellous bone. The large marrow spaces hyperkalemia is an adverse effect.
are reduced to haversian canals. 27. A. Albuterol’s sympathomimetic effect causes cardiac
9. A. Viscosity, a measure of a fluid’s internal resistance stimulation that may cause tachycardia and
to flow, is increased as the number of red cells palpitation.
suspended in plasma. 28. D. Changing positions slowly will help prevent the side
10. C. Hemiparesis creates instability. Using a cane effect of orthostatic hypotension.
provides a wider base of support and, therefore 29. A. Therapeutic effects of simvastatin include decreased
greater stability. serum triglyceries, LDL and cholesterol.
11. D. Manual stretching exercises will assist in keeping 30. C. Nitroglycerine is sensitive to light and moisture ad
the muscles and tendons supple and pliable, reducing must be stored in a dark, airtight container.
the traumatic consequences of repetitive activity. 31. A. Visual disturbance are a sign of toxicity because
12. C. The length of the urethra is shorter in females than retinopathy can occur with this drug.
in males; therefore microorganisms have a shorter 32. B. Toxic levels of Lanoxin stimulate the medullary
distance to travel to reach the bladder. The proximity chemoreceptor trigger zone, resulting in nausea and
of the meatus to the anus in females also increases this subsequent anorexia.
incidence. 33. B. Warfarin derivatives cause an increase in the
13. D. Temperature may increase within the first 24 hours prothrombin time and INR, leading to an increased risk
and persist as long as a week. for bleeding. Any abnormal or excessive bleeding must
14. C. The hips are in extension when the client is prone; be reported, because it may indicate toxic levels of the
this keeps the hips from flexing. drug.
15. C. Steroids have an anti-inflammatory effect that can 34. B. Levodopa is the metabolic precursor of dopamine. It
reduce arthritic pannus formation. reduces sympathetic outflow by limiting
16. A. Activities such as rigorous brushing of hair and teeth vasoconstriction, which may result in orthostatic
cause increased intraocular pressure and may lead to hypotension.
hemorrhage in the anterior chamber. 35. A. Tensilon, an anticholinesterase drug, causes
17. C. This decrease in PaO2 indicates respiratory failure; it temporary relief of symptoms of myasthenia gravis in
warrants immediate medical evaluation.
client who have the disease and is therefore an items, there is no problem to use it by other people. It is
effective diagnostic aid. important for the nurse to understand that
36. D. Carbamazepine ( Tegretol) is administered to psychodynamically, the behavior of the client may be largely
control pain by reducing the transmission of nerve attributed to a development defect related to the:
impulses in clients with trigeminal neuralgia.
37. C. Potassium iodide, which aids in decreasing the Oedipal complex
vascularity of the thyroid gland, decreases the risk for Superego
hemorrhage. Id
38. C. Hydrocortisone is a glucocorticoid that has anti- Ego
inflammatory action and aids in metabolism of 2. A client tells the nurse, “Yesterday, I was planning to kill
carbohydrate, fat, and protein, causing elevation of myself.” What is the best nursing response to this cient?
blood glucose. Thus it enables the body to adapt to
stress. “What are you going to do this time?”
39. D. DDAVP replaces the ADH, facilitating reabsorption Say nothing. Wait for the client’s next comment
of water and consequent return of normal urine “You seem upset. I am going to be here with you; perhaps
output and thirst. you will want to talk about it”
40. B. To prevent crystal formation, the client should have “Have you felt this way before?”
sufficient intake to produce 1000 to 1500 mL of urine 3. In crisis intervention therapy, which of the following
daily while taking this drug. principle that the nurse will use to plan her/his goals?
41. A. Prolonged chemotherapy may slow the production
of leukocytes in bone marrow, thus suppressing the Crises are related to deep, underlying problems
activity of the immune system. Antibiotics may be Crises seldom occur in normal people’s lives
required to help counter infections that the body can Crises may go on indefinitely.
no longer handle easily. Crises usually resolved in 4-6 weeks.
42. A. Any hormone normally produced by the body must 4. The nurse enters the room of the male client and found
be withdrawn slowly to allow the appropriate organ to out that the client urinates on the floor. The client hides
adjust and resume production. when the nurse is about to talk to him. Which of the
43. B. Dehydration is most readily and accurately following is the best nursing intervention?
measured by serial assessment of body weight; 1 L of
fluid weighs 2.2 pounds. Place restriction on the client’s activities when his behavior
44. A. The concentration of potassium is greater inside the occurs.
cell and is important in establishing a membrane Ask the client to clean the soiled floor.
potential, a critical factor in the cell’s ability to Take the client to the bathroom at regular intervals.
function. Limit fluid intake.
45. B. The kidneys normally eliminate potassium from the 5. A young lady with a diagnosis of schizophrenic reaction is
body; hyperkalemia may necessitate dialysis. admitted to the psychiatric unit. In the past two months, the
46. C. Vital signs monitor cardiorespiratory status; client has poor appetite, experienced difficulty in sleeping,
hyperkalemia causes serious cardiac dysrhythmias. was mute for long periods of time, just stayed in her room,
47. A. Once treatment with insulin for diabetic grinning and pointing at things. What would be the initial
ketoacidosis is begun, potassium ions reenter the cell, nursing action on admitting the client to the unit?
causing hypokalemia; therefore potassium, along with
the replacement fluid, is generally supplied. Assure the client that “ You will be well cared for.”
48. C. Potassium, the major intracellular cation, functions Introduce the client to some of the other clients.
with sodium and calcium to regulate neuromuscular Ask “Do you know where you are?”
activity and contraction of muscle fibers, particularly Take the client to the assigned room.
the heart muscle. In hypokalemia these symptoms 6. A 16-year-old girl was diagnosed with anorexia. What
develop. would be the first assessment of the nurse?
49. A. Because IV solutions enter the body’s internal
environment, all solutions and medications utilizing What food she likes.
this route must be sterile to prevent the introduction Her desired weight.
of microbes. Her body image.
50. A. A low pH and bicarbonate level are consistent with What causes her behavior.
metabolic acidosis. 7. On an adolescent unit, a nurse caring to a client was
informed that her client’s closest roommate dies at night.
PNLE V Nursing Practice What would be the most appropriate nursing action?
The scope of this Nursing Test V is parallel to the NP5 NLE
Coverage: Do not bring it up unless the client asks.
Psychiatric Nursing Tell the client that her roommate went home.
Tell the client, if asked, “You should ask the doctor.”
1. A 17-year-old client has a record of being absent in the Tell the client that her closest roommate died.
class without permission, and “borrowing” other people’s 8. A woman gave birth to an unhealthy infant, and with some
things without asking permission. The client denies stealing; body defects. The nurse should expect the woman’s initial
rationalizing instead that as long as no one was using the reactions to include:
15. The client is telling the nurse in the psychiatric ward, “I
Depression hate them.” Which of the following is the most appropriate
Withdrawal nursing response to the client?
Apathy
Anger “Tell me about your hate.”
9. A client in the psychiatric unit is shouting out loud and tells “I will stay with you as long as you feel this way.”
the nurse, “Please, help me. They are coming to get me.” “For whom do you have these feelings?”
What would be the appropriate nursing response? “I understand how you can feel this way.”
16. The mother visits her son with major depression in the
“ I won’t let anyone get you.” psychiatric unit. After the conversation of the client and the
“Who are they?” mother, the nurse asks the mother how it is talking to her
“I don’t see anyone coming.” son. The mother tells the nurse that it was a stressful time.
“You look frightened.” During an interview with the client, the client says, “we had a
10. A client who is severely obese tells the nurse, “My marvelous visit.” Which of the following coping mechanism
therapist told me that I eat a lot because I didn’t get any can be described to thestatement of the client?
attention and love from my mother. What does the therapist
mean?” What is the best nursing response? Identification.
Rationalization.
“What do you think is the connection between your not Denial.
getting enough love and overeating?” Compensation.
“Tell me what you think the therapist means.” 17. A male client is quiet when the physician told him that he
“You need to ask your therapist.” has stage IV cancer and has 4 months to live. The nurse
“ We are here to deal with your diet, not with your determines that this reaction may be an example of:
psychological problems.”
11. After the discussion about the procedure the physician Indifference
scheduled the client for mastectomy. The client tells the Denial
nurse, “If my breasts will be removed, I’m afraid my husband Resignation
will not love me anymore and maybe he will never touch Anger
me.” What should the nurse’s response? 18. A nurse is caring to a female client with five young
children. The family member told the client that her ex-
“I doubt that he feels that way.” husband has died 2 days ago. The reaction of the client is
“What makes you feel that way?” stunned silence, followed by anger that the ex-husband left
“Have you discussed your feelings with your husband?” no insurance money for their young children. The nurse
Ask the husband, in front of the wife, how he feels about this. should understand that:
12. The child is brought to the hospital by the parents. During
assessment of the nurse, what parental behavior toward a The children and the injustice done to them by their father’s
child should alert the nurse to suspect child abuse? death are the woman’s main concern.
To explain the woman’s reaction, the nurse needs more
Ignoring the child. information about the relationship and breakup.
Flat affect. The woman is not reacting normally to the news.
Expressions of guilt. The woman is experiencing a normal bereavement reaction.
Acting overly solicitous toward the child 19. A client who is manic comes to the outpatient
13. A nurse is caring to a client with manic disorder in the department. The nurse is assigning an activity for the client.
psychiatric ward. On the morning shift, the nurse is talking What activity is best for the nurse to encourage for a client in
with the client who is now exhibiting a manic episode with a manic phase?
flight of ideas. The nurse primarily needs to:
Solitary activity, such as walking with the nurse, to decrease
Focus on the feelings conveyed rather than the thoughts stimulation.
expressed. Competitive activity, such as bingo, to increase the client’s
Speak loudly and rapidly to keep the client’s attention, self-esteem.
because the client is easily distracted. Group activity, such as basketball, to decrease isolation.
Allow the client to talk freely. Intellectual activity, such as scrabble, to increase
Encourage the client to complete one thought at a time. concentration.
14. The nurse is caring to an autistic child. Which of the 20. The nurse is about to administer Imipramine HCI
following play behavior would the nurse expect to see in a (Tofranil) to the client, the client says, “Why should I take
child? this?” The doctor started me on this 10days ago; it didn’t
help me at all.” Which of the following is the best nursing
competitive play response:
nonverbal play
cooperative play “What were you expecting to happen?”
solitary play “It usually takes 2-3 weeks to be effective.”
“Do you want to refuse this medication? You have the right.”
“That’s a long time wait when you feel so depressed.”
21. Which of the following drugs the nurse should choose to 28. A female client tells the nurse that she is afraid to go out
administer to a client to prevent pseudoparkinsonism? from her room because she thinks that the other client might
kill her. The nurse is aware that this behavior is related to:
Isocarboxazid (Marplan)
Chlorpromazine HCI (Thorazine) Hallucination.
Trihexyphenidyl HCI (Artane) Ideas of reference.
Trifluoperazine HCI (Stelazine) Delusion of persecution.
22. The nurse is caring to an 80-year-old client with Illusion.
dementia? What is the most important psychosocial need for 29. A female client is taking Imipramine HCI (Tofranil) for
this client? almost 1 week and shows less awareness of the physical
body. What problem would the nurse be most concerned?
Focus on the there-and-then rather the here-and-now.
Limit in the number of visitors, to minimize confusion. Nausea.
Variety in their daily life, to decrease depression. Gait disturbances.
A structured environment, to minimize regressive behaviors. Bowel movements.
23. A client tells the nurse, “I don’t want to eat any meals Voiding.
offered in this hospital because the food is poisoned.” The 30. A 6-year-old client dies in the nursing unit. The parents
nurse is aware that the client is expressing an example of: want to see the child. What is the most appropriate nursing
action?
Delusion.
Hallucination. Give the parents time alone with the body.
Negativism. Ask the physician for permission.
Illusion. Complete the postmortem care and quietly accompany the
24. A client is admitted in the hospital. On assessment, the family to the child’s room.
nurse found out that the client had several suicidal attempts. Suggest the parents to wait until the funeral service to say
Which of the following is the most important nursing action? “good-bye.”
31. A 20-year-old female client is diagnosed with anxiety
Ignore the client as long as he or she is talking about suicide, disorder. The physician prescribed Flouxetine (Prozac). What
because suicide attempt is unlikely. is the most important side effects should a nurse be
Administer medication. concerned?
Relax vigilance when the client seems to be recovering from
depression. Tremor, drowsiness.
Maintain constant awareness of the client’s whereabouts. Seizures, suicidal tendencies.
25. The nurse suspects that the client is suffering from Visual disturbance, headache.
depression. During assessment, what are the most Excessive diaphoresis, diarrhea.
characteristic signs and symptoms of depression the nurse 32. A nurse is assigned to activate a client who is withdrawn,
would note? hears voices and negativistic. What would be the best nursing
approach?
Constipation, increased appetite.
Anorexia, insomnia. Mention that the “voices” would want the client to
Diarrhea, anger. participate.
Verbosity, increased social interaction. Demand that the client must join a group activity.
26. The client in the psychiatric unit states that, “The goodas Give the client a long explanation of the benefits of activity.
are coming! I must be ready.” In response to this neologism, Tell the client that the nurse needs a partner for an activity.
the nurse’s initial response is to: 33. A nurse is going to give a rectal suppository as a
preoperative medication to a 4-year-old boy. The boy is very
Acknowledge that the word has some special meaning for the anxious and frightened. Which of the following statement by
client. the nurse would be most appropriate to gain the child’s
Try to interpret what the client means. cooperation?
Divert the client’s attention to an aspect of reality.
State that what the client is saying has not been understood “Be a big kid! Everyone’s waiting for you.”
and then divert attention to something that is really bound. “Lie still now and I’ll let you have one of your presents before
27. A male client diagnosed with depression tells the nurse, “I you even have your operation.”
don’t want to look weak and I don’t even cry because my “Take a nice, big, deep breath and then let me hear you
wife and my kids can’t bear it.” The nurse understands that count to five.”
this is an example of: “You look so scared. Want to know a secret? This won’t hurt
a bit!”
Repression. 34. A depressed client is on an MAO inhibitor? What should
Suppression. the nurse watch out for?
Undoing.
Rationalization. Hypertensive crisis.
Diet restrictions.
Taking medication with meals.
Exposure to sunlight. Maintain a therapeutic environment.
35. A 16-year-old girl is admitted for treatment of a fracture. 41. A 3-year-old boy is brought to the emergency
The client shares to the nurse caring to her that her step- department. After an hour, the boy dies of respiratory failure.
father has made sexual advances to her. She got the chance The mother of the boy becomes upset, shouting and abusive,
to tell it to her mother but refuses to believe. What is the saying to the nurse, “If it had been your son, they would have
most therapeutic action of the nurse would be: done more to save it. “What should the nurse say or do?

Tell the client to work it out with her father. Touch her and tell her exactly what was done for her baby.
Tell the client to discuss it with her mother. Allow the mother to continue her present behavior while
Ask the father about it. sitting quietly with her.
Ask the mother what she thinks. “No, all clients are given the same good care.”
36. A client with a diagnosis of paranoid disorder is admitted “Yes, you’re probably right. Your son did not get better care.”
in the psychiatric hospital. The client tells the nurse, “the FBI 42. The nurse is interacting to a client with an antisocial
is following me. These people are plotting against me.” With personality disorder. What would be the most therapeutic
this statement the nurse will need to: approach of the nurse to an antisocial behavior?

Acknowledge that this is the client’s belief but not the nurse’s Gratify the client’s inner needs.
belief. Give the client opportunities to test reality.
Ask how that makes the client feel. Provide external controls.
Show the client that no one is behind. Reinforce the client’s self-concept.
Use logic to help the client doubt this belief. 43. A 55-year-old male client tells the nurse that he needs his
37. A nurse is completing the routine physical examination to glasses and hearing aid with him in the recovery room after
a healthy 16-year-old male client. The client shares to the the surgery, or he will be upset for not granting his request.
nurse that he feels like killing his girlfriend because he found What is the appropriate nursing response?
out that her girlfriend had another boyfriend. He then laughs,
and asks the nurse to keep this a secret just between the two “Do you get upset and confused often?”
of them. The nurse reviews his chart and notes that there is “You won’t need your glasses or hearing aid. The nurses will
no previously history of violence or psychiatric illness. Which take care of you.”
of the following would be the best action of the nurse to take “I understand. You will be able to cooperate best if you know
at this time? what is going on, so I will find out how I can arrange to have
your glasses and hearing aid available to you in the recovery
Suggest the teen meet with a counselor to discuss his feelings room.”
about his girlfriend. I understand you might be more cooperative if you have your
Tell the teen that his feelings are normal, and recommend aid and glasses, but that is just not possible. Rules, you
that he find another girlfriend to take his mind off the know.”
problem. 44. The male client had fight with his roommates in the
Recall the teenage boys often say things they really do not psychiatric unit. The client agitated client is placed in
mean and ignore the comment. isolation for seclusion. The nurse knows it is essential that:
Regard the comment seriously and notify the teen’s primary
health care provider and parents A staff member has frequent contacts with the client.
38. Which of the following person will be at highest risk for Restraints are applied.
suicide? The client is allowed to come out after 4 hours.
All the furniture is removed form the isolation room.
A student at exam time 45. A medical representative comes to the hospital unit for
A married woman, age 40, with 6 children. the promotion of a new product. A female client, admitted
A person who is an alcoholic. for hysterical behavior, is found embracing him. What should
A person who made a previous suicide attempt. the nurse say?
39. A male client is repetitively doing the handwashing every
time he touches things. It is important for a nurse to “Have you considered birth control?”
understand that the client’s behavior is probably an attempt “This isn’t the purpose of either of you being here.”
to: “I see you’ve made a new friend.”
“Think about what you are doing.”
Seek attention from the staff. 46. A client with dementia is for discharge. The nurse is
Control unacceptable impulses or feelings. providing a discharge instruction to the family member
Do what the voices the patient hears tell him or her to do. regarding safety measures at home. What suggestion can the
Punish himself or herself for guilt feeling. nurse make to the family members?
40. In a mental health settings, the basic goal of nursing is to:
Avoid stairs without banisters.
Advance the science of psychiatry by initiating research and Use restraints while the client is in bed to keep him or her
gathering data for current statistics on emotional illness. from wandering off during the night.
Plan activity programs for clients. Use restraints while the client is sitting in a chair to keep him
Understand various types of family therapy and psychological or her from wandering off during the day.
tests and how to interpret them. Provide a night-light and a big clock.
47. A 30-year-old married woman comes to the hospital for 8. D. The woman is experiencing an actual loss and will
treatment of fractures. The woman tells the nurse that she probably exhibit many of the same symptoms as a
was physically abused by her husband. The woman receives a person who has lost someone to death.
call from her husband telling her to get home and things will 9. C. This option is an example of pointing out reality- the
be different. He felt sorry of what he did. What can the nurse nurse’s perception.
advise her? 10. B. This response asks information that the nurse can
use. If the client understands the statement, the nurse
“Do you think so?” can support the therapist when focusing on connection
“It’s not likely.” between food, love, and mother. If the client does not
“What will be different?” understand thestatement, the nurse can help get
“I hope so, for your sake.” clarification from the therapist.
48. A female client was diagnosed with breast cancer. It is 11. C. This option redirects the client to talk to her
found to be stage IV, and a modified mastectomy is husband.
performed. After the procedure, what behaviors could the 12. D. This is an example of reaction formation, a coping
nurse expects the client to display? mechanism.
13. A. Often the verbalized ideas are jumbled, but the
Denial of the possibility of carcinoma. underlying feelings are discernible and must be
Signs of grief reaction. acknowledged.
Relief that the operation is over. 14. D. Autistic children do best with solitary play because
Signs of deep depression. they typically do not interact with others in a socially
49. A client is withdrawn and does not want to interact to comprehensible and acceptable way.
anybody even to the nurse. What is the best initial nursing 15. A. The nurse is asking the client to clarify and further
approach to encourage communication with this client? discuss feelings.
16. C. Denial is the act of avoiding disagreeable realities by
Use simple questions that call for a response. ignoring them.
Encourage discussion of feelings. 17. B. Reactions when told of a life-threatening illness
Look through a photo album together. stem from Kübler-Ross’ ideas on death and dying.
Bring up neutral topics. Denial is a typical grief response, and usually is a first
50. Which of the following nursing approach is most reaction.
important in a client with depression? 18. D. Shock and anger are commonly the primary initial
reactions.
Deemphasizing preoccupation with elimination, nourishment, 19. A. This option avoids external stimuli, yet channels the
and sleep. excess motor activity that is often part of the manic
Protecting against harm to others. phase.
Providing motor outlets for aggressive, hostile feelings. 20. B. The patient needs a brief, factual answer.
Reducing interpersonal contacts. 21. C. Trihexyphenidyl HCI (Artane) is often used to
counteract side effect of pseudoparkinsonism, which
Answers and Rationales often accompanies the use of phenothiazine, such as
1. B. This shows a weak sense of moral consciousness. chlorpromazine HCI (Thorazine or Trifluoperazine HCI
According to Freudian theory, personality disorders (Stelazine).
stem from a weak superego. 22. D. Persons with dementia needs sameness,
2. C. The client needs to have his or her feelings consistency, structure, routine, and predictability.
acknowledged, with encouragement to discuss 23. A. This is a false belief developed in response to an
feelings, and be reassured about the nurse’s presence. emotional need.
3. D. Part of the definition of a crisis is a time span of 4-6 24. D. The client must be constantly observed.
weeks. 25. B. The appetite is diminished and sleeping is affected
4. C. The client is most likely confused, rather than to a client with depression.
exhibiting acting-out, hostile behavior. Frequent 26. A. It is important to acknowledge a statement, even if
toileting will allow urination in an appropriate place. it is not understood.
5. D. The client needs basic, simple orientation that 27. D. Rationalization is the process of constructing
directly relates to the here-and-now, and does not plausible reasons for one’s responses.
require verbal interaction. 28. C. The client has ideas that someone is out to kill her.
6. A. Although all options may appear correct. A is the 29. D. A serious side effect of Imipramine HCI (Tofranil) is
best because it focuses on a range of possible positive urinary retention (voiding problems)
reinforcers, a basis for an effective behavior 30. A. This allows the parents/family to grieve over the
modification program. It can lead to concrete, specific loss of the child, by going through the steps of leave
nursing interventions right away and provides a taking.
therapeutic use of “control” for the 16-year-old. 31. B. Assess for suicidal tendencies, especially during
7. A. The nurse needs to wait and see: do not “jump the early therapy. There is an increased risk of seizures in
gun”; do not assume that the client wants to know debilitated client and those with a history of seizures.
now. 32. D. The nurse helps to activate by doing something with
the client.
33. C. Preschool children commonly experience fears and
fantasies regarding invasive procedures. The nurse
should attempts to momentarily distract the child with
a simple task that can be easily accomplished while the
child remains in the side-lying position. The
suppository can be slipped into place while the child is
counting, and then the nurse can praise the child for
cooperating, while holding the buttocks together to
prevent expulsion of the suppository.
34. A. This is the more inclusive answer, although diet
restrictions (answer1) are important, their purpose is
to prevent hypertensive crisis (answer 2).
35. D. This comes closest to beginning to focus on family-
centered approach to intervene in the “conspiracy of
silence”. This is therefore the best among the options.
36. A. The nurse should neither challenge nor use logic to
dispel an irrational belief.
37. D. Any threat to the safety of oneself or other should
always be taken seriously and never disregarded by
the nurse.
38. C. The likelihood of multiple contributing factors may
make this person at higher risk for suicide. Some
factors that may exist are physical illness related to
alcoholism, emotional factors ( anxiety, guilt,
remorse), social isolation due to impaired relationships
and economic problems related to employment.
39. B. A ritual, such as compulsive handwashing, is an
attempt to allay anxiety caused by unconscious
impulses that are frightening.
40. D. This is the most neutral answer by process of
elimination.
41. B. This option allows a normal grief response (anger).
42. C. Personality disorders stem from a weak superego,
implying a lack of adequate controls.
43. C. The client will be easier to care for if he has his
hearing aid and glasses.
44. A. Frequent contacts at times of stress are important,
especially when a client is isolated.
45. B. This response is aimed at redirecting the
inappropriate behavior.
46. D. This option is best to decrease confusion and
disorientation to place and time.
47. C. This option helps the woman to think through and
elaborate on her own thoughts and prognosis.
48. B. It is mostly likely that grief would be expressed
because of object loss.
49. D. Neutral, nonthreatening topics are best in
attempting to encourage a response.
50. C. It is important to externalize the anger away from
self.

You might also like